Medsurg 3

You might also like

Download as pdf or txt
Download as pdf or txt
You are on page 1of 71

1.1 Cardiac Failure complex measures 0.08 second.

The nurse
​1.2 Acute myocardial infarction interprets this rhythm is:
1. A nurse caring for a client in one room is told by A. Normal sinus rhythm
another nurse that a second client has developed B. Sinus bradycardia
severe pulmonary edema. On entering the 2nd C. Sinus tachycardia
client’s room, the nurse would expect the client to D. Sinus dysrhythmia
be:
A. Slightly anxious 6. A client has frequent bursts of ventricular
B. Mildly anxious tachycardia on the cardiac monitor. A nurse is most
C. Moderately anxious concerned with this dysrhythmia because:
D. Extremely anxious
A. It is uncomfortable for the client, giving a sense
2. A client with pulmonary edema has been on of impending doom.
diuretic therapy. The client has an order for B. It produces a high cardiac output that quickly
additional furosemide (Lasix) in the amount of 40 leads to cerebral and myocardial ischemia.
mg IV push. Knowing that the client also will be C. It is almost impossible to convert to a normal
started on digoxin (Lanoxin), a nurse checks the sinus rhythm.
client’s most recent: D. It can develop into ventricular fibrillation at any
A. Digoxin level time.
B. Sodium level
C. Potassium level 7. A home care nurse is making a routine visit to a
D. Creatinine level client receiving digoxin (Lanoxin) in the treatment of
heart failure. The nurse would particularly assess
3. A client who had cardiac surgery 24 hours ago the client for:
has a urine output averaging 19 ml/hr for 2 hours. A. Thrombocytopenia and weight gain
The client received a single bolus of 500 ml of IV B. Anorexia, nausea, and visual disturbances
fluid. Urine output for the subsequent hour was 25 C. Diarrhea and hypotension
ml. Daily laboratory results indicate the blood urea D. Fatigue and muscle twitching
nitrogen is 45 mg/dL and the serum creatinine is
2.2 mg/dL. A nurse interprets the client is at risk for: 8. A client with angina complains that the angina
A. Hypovolemia pain is prolonged and severe and occurs at the
B. UTI same time each day, most often in the morning, On
C. Glomerulonephritis further assessment a nurse notes that the pain
D. Acute renal failure occurs in the absence of precipitating factors. This
type of anginal pain is best described as:
4. A nurse is preparing to ambulate a client on the A. Stable angina
3rd day after cardiac surgery. The nurse would plan B. Unstable angina
to do which of the following to enable the client to C. Variant angina
best tolerate the ambulation? D. Nonanginal pain
A. Encourage the client to cough and deep
breathe. 9. The physician orders continuous intravenous
B. Premedicate the client with an analgesic. nitroglycerin infusion for the client with MI. Essential
C. Provide the client with a walker. nursing actions include which of the following?
D. Remove telemetry equipment because it weighs A. Obtaining an infusion pump for the medication.
down the hospital gown. B. Monitoring BP q4h.
C. Monitoring urine output hourly.
5. A client’s electrocardiogram strip shows atrial D. Obtaining serum potassium levels daily.
and ventricular rates of 80 complexes per minute.
The PR interval is 0.14 second, and the QRS
10. Aspirin is administered to the client E. Acute pain related to inability to meet the
experiencing an MI because of its: oxygen demands.
A. Antipyretic action
B. Antithrombotic action 15. Which of the following would be a priority
C. Antiplatelet action nursing diagnosis for the client with heart failure
D. Analgesic action and pulmonary edema?
A. Risk for infection related to stasis of alveolar
11. Which of the following is an expected outcome secretions.
for a client on the second day of hospitalization B. Impaired skin integrity related to pressure.
after an MI? C. Activity intolerance related to pump failure.
A. Has severe chest pain. D. Constipation related to immobility.
B. Can identify risk factors for MI.
C. Agrees to participate in a cardiac rehabilitation 16. Captopril may be administered to a client with
walking program. HF because it acts as a:
D. Can perform personal self-care activities without A. Vasopressor
pain. B. Volume expander
C. Vasodilator
12. Which of the following reflects the principle on D. Potassium-sparing diuretic
which a client’s diet will most likely be based during
the acute phase of MI? 17. Furosemide is administered intravenously to a
client with HF. How soon after administration should
A. Liquids as ordered the nurse begin to see evidence of the drug’s
B. Small, easily digested meals desired effect?
C. Three regular meals per day A. 5 to 10 minutes
D. NPO B. 30 to 60 minutes
Incorrect C. 2 to 4 hours
D. 6 to 8 hours
13. An older, sedentary adult may not respond to
emotional or physical stress as well as a younger 18. Which of the following foods should the nurse
individual because of: teach a client with heart failure to avoid or limit
A. Left ventricular atrophy when following a 2-gram sodium diet?
B. Irregular heartbeats A. Apples
C. Peripheral vascular occlusion B. Tomato juice
D. Pacemaker placement C. Whole wheat bread
D. Beef tenderloin
14. Question1 point(s)
Which of the following nursing diagnoses would be 19. The nurse finds the apical pulse below the 5th
appropriate for a client with heart failure? Select all intercostal space. The nurse suspects:
that apply. A. Left atrial enlargement
A. Ineffective tissue perfusion related to decreased B. Left ventricular enlargement
peripheral blood flow secondary to decreased C. Right atrial enlargement
cardiac output. D. Right ventricular enlargement
B. Activity intolerance related to increased cardiac
output. 20. Which of the following arteries primarily feeds
C. Decreased cardiac output related to structural the anterior wall of the heart?
and functional changes. A. Circumflex artery
D. Impaired gas exchange related to decreased B. Internal mammary artery
sympathetic nervous system activity. C. Left anterior descending artery
D. Right coronary artery
A. Dilated aorta.
21. The most important factor in regulating the B. Normally functioning heart.
caliber of blood vessels, which determines C. Decreased myocardial contractility.
resistance to flow, is: D. Failure of the ventricle to eject all of the blood
A. Hormonal secretion during systole.
B. Independent arterial wall activity.
C. The influence of circulating chemicals 3. Which of the following classes of drugs is most
D. The sympathetic nervous system widely used in the treatment of cardiomyopathy?
A. Antihypertensives
22. With peripheral arterial insufficiency, leg pain B. Beta-adrenergic blockers
during rest can be reduced by: C. Calcium channel blockers
A. Elevating the limb above heart level. D. Nitrates
B. Lowering the limb so it is dependent.
C. Massaging the limb after application of cold 4. If medical treatments fail, which of the following
compresses. invasive procedures is necessary for treating
D. Placing the limb in a plane horizontal to the cardiomyopathy?
body. A. Cardiac catheterization
B. Coronary artery bypass graft (CABG)
23. Buerger’s disease is characterized by all of the C. Heart transplantation
following except: D. Intra-aortic balloon pump (IABP)
A. Arterial thrombosis formation and occlusion.
B. Lipid deposits in the arteries. 5. Which of the following conditions is associated
C. Redness or cyanosis in the limb when it is with a predictable level of pain that occurs as a
dependent. result of physical or emotional stress?
D. Venous inflammation and occlusion. A. Anxiety
B. Stable angina
24. A significant cause of venous thrombosis is: C. Unstable angina
A. Altered blood coagulation D. Variant angina
B. Stasis of blood
C. Vessel wall injury 6. Which of the following types of angina is most
D. All of the above closely related with an impending MI?
Incorrect A. Angina decubitus
25. When caring for a patient who has started B. Chronic stable angina
anticoagulant therapy with warfarin (Coumadin), the C. Nocturnal angina
nurse knows not to expect therapeutic benefits for: D. Unstable angina
A. At least 12 hours
B. The first 24 hours 7. Which of the following conditions is the
C. 2-3 days predominant cause of angina?
D. 1 week A. Increased preload
B. Decreased afterload
1. In which of the following types of cardiomyopathy C. Coronary artery spasm
does cardiac output remain normal? D. Inadequate oxygen supply to the myocardium.
A. Dilated
B. Hypertrophic 8. Which of the following tests is used most often to
C. Obliterative diagnose angina?
D. Restrictive A. Chest x-ray
B. Echocardiogram
2. Which of the following cardiac conditions does a C. Cardiac catheterization
fourth heart sound (S4) indicate? D. 12-lead electrocardiogram (ECG)
15. Which of the following actions is the appropriate
9. Which of the following results is the primary initial response to a client coughing up pink, frothy
treatment goal for angina? sputum?
A. Call for help.
A. Reversal of ischemia. B. Call the physician.
B. Reversal of infarction. C. Start an I.V. line.
C. Reduction of stress and anxiety. D. Suction the client.
D. Reduction of associated risk factors.
16. Which of the following terms describes the force
10. Which of the following interventions should be against which the ventricle must expel blood?
the first priority when treating a client experiencing A. Afterload
chest pain while walking? B. Cardiac output
A. Sit the client down. C. Overload
B. Get the client back to bed. D. Preload
C. Obtain an ECG.
D. Administer sublingual nitroglycerin. 17. Acute pulmonary edema caused by heart failure
is usually a result of damage to which of the
11. Myocardial oxygen consumption increases as following areas of the heart?
which of the following parameters increase? A. Left atrium
A. Preload, afterload, and cerebral blood flow. B. Right atrium
B. Preload, afterload, and renal blood flow. C. Left ventricle
C. Preload, afterload, contractility, and heart rate. D. Right ventricle
D. Preload, afterload, cerebral blood flow, and
heart rate. 18. An 18-year-old client who recently had an URI
is admitted with suspected rheumatic fever. Which
12. Which of the following positions would best aid assessment findings confirm this diagnosis?
breathing for a client with acute pulmonary edema? A. Erythema marginatum, subcutaneous nodules,
A. Lying flat in bed. and fever
B. Left side-lying. B. Tachycardia, finger clubbing, and a load S3
C. In high Fowler’s position. C. Dyspnea, cough, and palpitations
D. In semi-Fowler’s position. D. Dyspnea, fatigue, and syncope

13. Which of the following blood gas abnormalities 19. A client admitted with angina complains of
is initially most suggestive of pulmonary edema? severe chest pain and suddenly becomes
unresponsive. After establishing unresponsiveness,
A. Anoxia which of the following actions should the nurse take
B. Hypercapnia first?
C. Hyperoxygenation A. Activate the resuscitation team.
D. Hypocapnia B. Open the client’s airway.
C. Check for breathing.
14. Which of the following is a compensatory D. Check for signs of circulation.
response to decreased cardiac output?
A. Decreased BP. 20. A 55-year-old client is admitted with an acute
B. Alteration in LOC. inferior-wall myocardial infarction. During the
C. Decreased BP and diuresis. admission interview, he says he stopped taking his
D. Increased BP and fluid retention. metoprolol (Lopressor) 5 days ago because he was
feeling better. Which of the following nursing
diagnoses takes priority for this client?
A. Anxiety
B. Ineffective tissue perfusion; cardiopulmonary C. Peptic ulcer disease
C. Acute pain D. A-Fib
D. Ineffective therapeutic regimen management
25. A nurse is preparing for the admission of a
21. A client comes into the E.R. with acute client with heart failure who is being sent directly to
shortness of breath and a cough that produces the hospital from the physician’s office. The nurse
pink, frothy sputum. Admission assessment reveals would plan on having which of the following
crackles and wheezes, a BP of 85/46, a HR of 122 medications readily available for use?
BPM, and a respiratory rate of 38 breaths/minute. A. diltiazem (Cardizem)
The client’s medical history included DM, HTN, and B. digoxin (Lanoxin)
heart failure. Which of the following disorders C. propranolol (Inderal)
should the nurse suspect? D. metoprolol (Lopressor)
A. Pulmonary edema
B. Pneumothorax 1. Mike, a 43-year old construction worker, has a
C. Cardiac tamponade history of hypertension. He smokes two packs of
D. Pulmonary embolism cigarettes a day, is nervous about the possibility of
being unemployed, and has difficulty coping with
22. The nurse coming on duty receives the report stress. His current concern is calf pain during
from the nurse going off duty. Which of the following minimal exercise that decreased with rest. The
clients should the on-duty nurse assess first? nurse assesses Mike’s symptoms as being
A. The 58-year-old client who was admitted 2 days associated with peripheral arterial occlusive
ago with heart failure, BP of 126/76, and a disease. The nursing diagnosis is probably:
respiratory rate of 21 breaths a minute. A. Alteration in tissue perfusion related to
B. The 88-year-old client with end-stage right-sided compromised circulation.
heart failure, BP of 78/50, and a DNR order. B. Dysfunctional use of extremities related to
C. The 62-year-old client who was admitted one muscle spasms.
day ago with thrombophlebitis and receiving IV C. Impaired mobility related to stress associated
heparin. with pain.
D. A 76-year-old client who was admitted 1 hour D. Impairment in muscle use is associated with
ago with new-onset atrial fibrillation and is receiving pain on exertion.
IV diltiazem (Cardizem).
2. A 24-year old man seeks medical attention for
23. When developing a teaching plan for a client complaints of claudication in the arch of the foot. A
with endocarditis, which of the following points is nurse also notes superficial thrombophlebitis of the
most essential for the nurse to include? lower leg. The nurse would next assess the client
A. “Report fever, anorexia, and night sweats to the for:
physician.” A. Familial tendency toward peripheral vascular
B. “Take prophylactic antibiotics after dental work disease.
and invasive procedures.” B. Smoking history.
C. “Include potassium rich foods in your diet.” C. Recent exposures to allergens.
D. “Monitor your pulse regularly.” D. History of insect bites.

24. A nurse is conducting a health history with a 3. Intravenous heparin therapy is ordered for a
client with a primary diagnosis of heart failure. client. While implementing this order, a nurse
Which of the following disorders reported by the ensures that which of the following medications is
client is unlikely to play a role in exacerbating the available in the nursing unit?
heart failure? A. Vitamin K
A. Recent URI B. Aminocaproic acid
B. Nutritional anemia C. Potassium chloride
D. Protamine sulfate 7. A client is admitted with a venous stasis leg
ulcer. A nurse assesses the ulcer, expecting to note
4. A client who has been receiving heparin therapy that the ulcer:
also is started on warfarin sodium (coumadin). The A. Has a pale colored base.
client asks the nurse why both medications are B. Is deep, with even edges.
being administered. In formulating a response, the C. Has little granulation tissue.
nurse incorporates the understanding that warfarin D. Has brown pigmentation around it.
sodium:
A. Stimulates the breakdown of specific clotting 8. In preparation for the discharge of a client with
factors by the liver, and it takes 2-3 days for this to arterial insufficiency and Raynaud’s disease, client
exhibit an anticoagulant effect. teaching instructions should include:
B. Inhibits synthesis of specific clotting factors in A. Walking several times each day as an exercise
the liver, and it takes 3 to 4 days for this medication program.
to exert an anticoagulation effect. B. Keeping the heat up so that the environment is
C. Stimulates production of the body’s own warm.
thrombolytic substances, but it takes 2-4 days for it C. Wearing a TED hose during the day.
to begin. D. Using hydrotherapy for increasing oxygenation.
D. Has the same mechanism action of heparin,
and the crossover time is needed for the serum 9. A client comes to the outpatient clinic and tells
level of warfarin sodium to be therapeutic. the nurse that he has had leg pains that begin
when he walks but cease when he stops walking.
5. A nurse has the order to begin administering Which of the following conditions would the nurse
warfarin sodium (Coumadin) to a client. While assess for?
implementing this order, the nurse ensures that A. An acute obstruction in the vessels of the legs.
which of the following medications is available on B. Peripheral vascular problems in both legs.
the nursing unit as the antidote for Coumadin? C. Diabetes
A. Vitamin K D. Calcium deficiency
B. Aminocaproic acid
C. Potassium chloride 10. Which of the following characteristics is typical
D. Protamine sulfate of the pain associated with DVT?

6. A nurse is assessing the neurovascular status of A. Dull ache


a client who returned to the surgical nursing unit 4 B. No pain
hours ago after undergoing an aortoiliac bypass C. Sudden onset
graft. The affected leg is warm, and the nurse notes D. Tingling
redness and edema. The pedal pulse is palpable
and unchanged from admission. The nurse 11. Cancer can cause changes in what component
interprets that the neurovascular status is: of Virchow’s triad?
A. Normal because of the increased blood flow A. Blood coagulability
through the leg. B. Vessel walls
B. Slightly deteriorating and should be monitored C. Blood flow
for another hour. D. Blood viscosity
C. Moderately impaired, and the surgeon should
be called. 12. Varicose veins can cause changes in what
D. Adequate from the arterial approach, but component of Virchow’s triad?
venous complications are arising. A. Blood coagulability
B. Vessel walls
C. Blood flow
D. Blood viscosity
D. Chest pain
13. Which technique is considered the gold
standard for diagnosing DVT? 18. The nurse expects that a client with mitral
A. Ultrasound imaging stenosis would demonstrate symptoms associated
B. Venography with congestion in the:
C. MRI A. Aorta
D. Doppler flow study B. Right atrium
C. Superior vena cava
14. A nurse is assessing a client with an abdominal D. Pulmonary circulation
aortic aneurysm. Which of the following
assessment findings by the nurse is probably 19. Because a client has mitral stenosis and is a
unrelated to an aneurysm? prospective valve recipient, the nurse
A. Pulsatile abdominal mass. preoperatively assesses the client’s past
B. Hyperactive bowel sounds in that area. compliance with medical regimens. Lack of
C. Systolic bruit over the area of the mass. compliance with which of the following regimens
D. Subjective sensation of “heart beating” in the would pose the greatest health hazard to this
abdomen. client?
A. Medication therapy
15. A nurse is caring for a client who had a B. Diet modification
percutaneous insertion of an inferior vena cava C. Activity restrictions
filter and was on heparin therapy before surgery. D. Dental care
The nurse would inspect the surgical site most
closely for signs of: 20. Good dental care is an important measure in
A. Thrombosis and infection reducing the risk of endocarditis. A teaching plan to
B. Bleeding and infection promote good dental care in a client with mitral
C. Bleeding and wound dehiscence. stenosis should include a demonstration of the
D. Wound dehiscence and evisceration. proper use of:
A. A manual toothbrush
16. A 68-year-old woman is scheduled to undergo B. An electric toothbrush
mitral valve replacement for severe mitral stenosis C. An irrigation device
and mitral regurgitation. Although the diagnosis D. Dental floss
was made during childhood, she did not have any
symptoms until 4 years ago. Recently, she noticed 21. A client has been admitted to the hospital with a
increased symptoms, despite daily doses of digoxin diagnosis of suspected bacterial endocarditis. The
and furosemide. During the initial interview with the complication the nurse will constantly observe for
nice lady, the nurse would most likely learn that the is:
client’s childhood health history included: A. Presence of heart murmur
B. Systemic emboli
A. Chicken pox C. Fever
B. Poliomyelitis D. Congestive heart failure
C. Rheumatic fever
D. Meningitis 22. Cholesterol, frequently discussed in relation to
atherosclerosis, is a substance that:
17. Which of the following signs and symptoms
would most likely be found in a client with mitral A. May be controlled by eliminating food sources.
regurgitation? B. Is found in many foods, both plant and animal
A. Exertional dyspnea sources.
B. Confusion C. All persons would be better off without because
C. Elevated creatine phosphokinase concentration it causes the disease process.
D. Circulates in the blood, the level of which E. A 58 year old female with uncontrolled
usually decreases when unsaturated fats are hypertension and is being treated for influenza.
substituted for saturated fats.
2. A patient is being discharged home after
23. When teaching a client with a cardiac problem, hospitalization of left ventricular systolic
who is on a high-unsaturated fatty-acid diet, the dysfunction. As the nurse providing discharge
nurse should stress the importance of increasing teaching to the patient, which statement is NOT a
the intake of: correct statement about this condition?
A. Enriched whole milk A. "Signs and symptoms of this type of heart failure
B. Red meats, such as beef can include: dyspnea, persistent cough, difficulty
C. Vegetables and whole grains breathing while lying down, and weight gain."
D. Liver and other glandular organ meats B. "It is important to monitor your daily weights, fluid
and salt intake."
24. A 2-gram sodium diet is prescribed for a client C. "Left-sided heart failure can lead to right-sided
with severe hypertension. The client does not like heart failure, if left untreated."
the diet, and the nurse hears the client’s request D. "This type of heart failure can build up pressure
that the spouse “Bring in some good home-cooked in the hepatic veins and cause them to become
food.” It would be most effective for the nurse to congested with fluid which leads to peripheral
plan to: edema."
A. Call in the dietician for client teaching.
B. Wait for the client’s family and discuss the diet 3. Which of the following are NOT typical signs and
with the client and family. symptoms of right-sided heart failure?
C. Tell the client that the use of salt is forbidden, Select-all-that-apply:
because it will raise BP. A. Jugular venous distention
D. Catch the family members before they go into B. Persistent cough
the client’s room and tell them about the diet. C. Weight gain
D. Crackles
25. What criteria should the nurse use to determine E. Nocturia
normal sinus rhythm for a client on a cardiac F. Orthopnea
monitor? Select all that apply.
A. The RR intervals are relatively consistent. 4. A patient is diagnosed with left-sided systolic
B. One P wave precedes each QRS complex. dysfunction heart failure. Which of the following are
C. Four to eight complexes occur in a 6-second expected findings with this condition?
strip. A. Echocardiogram shows an ejection fraction of
D. The ST segment is higher than the PR interval. 38%.
E. The QRS complex ranges from 0.12 to 0.2 B. Heart catheterization shows an ejection fraction
seconds. of 65%.
C. Patient has frequent episodes of nocturnal
1. Which of the following patients are MOST at risk paroxysmal dyspnea.
for developing heart failure? Select-all-that-apply: D. Options A and C are both expected findings with
A. A 69 year old male with a history of alcohol left-sided systolic dysfunction heart failure.
abuse and is recovering from a myocardial
infarction. 5. True or False: Patients with left-sided diastolic
B. A 55 year old female with a health history of dysfunction heart failure usually have a normal
asthma and hypoparathyroidism. ejection fraction.
C. A 30 year old male with a history of endocarditis True
and has severe mitral stenosis. False
D. A 45 year old female with lung cancer stage 2.
6. A patient has a history of heart failure. Which of 10. What type of heart failure does this statement
the following statements by the patient indicates the describe? The ventricle is unable to properly fill with
patient may be experiencing heart failure blood because it is too stiff. Therefore, blood backs
exacerbation? up into the lungs causing the patient to experience
A. “I’ve noticed that I’ve gain 6 lbs in one week.” shortness of breath.
B. “While I sleep I have to prop myself up with a A. Left ventricular systolic dysfunction
pillow so I can breathe.” B. Left ventricular ride-sided dysfunction
C. “I haven’t noticed any swelling in my feet or C. Right ventricular diastolic dysfunction
hands lately.” D. Left ventricular diastolic dysfunction
D. Options B and C are correct.
E. Options A and B are correct. 11. A patient with left-sided heart failure is having
F. Options A, B, and C are all correct. difficulty breathing. Which of the following is the
most appropriate nursing intervention?
7. Patients with heart failure can experience A. Encourage the patient to cough and deep
episodes of exacerbation. All of the patients below breathe.
have a history of heart failure. Which of the B. Place the patient in Semi-Fowler's position.
following patients are at MOST risk for heart failure C. Assist the patient into High Fowler's position.
exacerbation? D. Perform chest percussion therapy.
A. A 55 year old female who limits sodium and fluid
intake regularly. 12. You're providing diet discharge teaching to a
B. A 73 year old male who reports not taking patient with a history of heart failure. Which of the
Amiodarone for one month and is experiencing following statements made by the patient
atrial fibrillation. represents they understood the diet teaching?
C. A 67 year old female who is being discharged A. "I will limit my sodium intake to 5-6 grams a day."
home from heart valve replacement surgery. B. "I will be sure to incorporate canned vegetables
D. A 78 year old male who has a health history of and fish into my diet."
eczema and cystic fibrosis. C. "I'm glad I can still eat sandwiches because I
love bologna and cheese sandwiches."
8. A 74 year old female presents to the ER with D. "I will limit my consumption of frozen meals."
complaints of dyspnea, persistent cough, and
unable to sleep at night due to difficulty breathing. 13. Select all the correct statements about
On assessment, you note crackles throughout the educating the patient with heart failure:
lung fields, respiratory rate of 25, and an oxygen A. It is important patients with heart failure notify
saturation of 90% on room air. Which of the their physician if they gain more than 6 pounds in a
following lab results confirm your suspicions of day or 10 pounds in a week.
heart failure? B. Patients with heart failure should receive an
A. K+ 5.6 annual influenza vaccine and be up-to-date with the
B. BNP 820 pneumonia vaccine.
C. BUN 9 C. Heart failure patients should limit sodium intake
D. Troponin <0.02 to 2-3 grams per day.
D. Heart failure is exacerbated by illness, too much
9. Which of the following tests/procedures are NOT fluid or sodium intake, and arrhythmias.
used to diagnose heart failure? E. Patients with heart failure should limit exercise
A. Echocardiogram because of the risks.
B. Brain natriuretic peptide blood test
C. Nuclear stress test 14. A patient taking Digoxin is experiencing severe
D. Holter monitoring bradycardia, nausea, and vomiting. A lab draw
shows that their Digoxin level is 4 ng/mL. What
medication do you anticipate the physician to order 19. Select all the correct statements about the
for this patient? pharmacodynamics of Beta-blockers for the
A. Narcan treatment of heart failure:
B. Aminophylline A. These drugs produce a negative inotropic effect
C. Digibind on the heart by increasing myocardial contraction.
D. No medication because this is a normal Digoxin B. A side effect of these drugs include bradycardia.
level. C. These drugs are most commonly prescribed for
patients with heart failure who have COPD.
15. Which of the following is a late sign of heart D. Beta-blockers are prescribed with ACE or ARBs
failure? to treat heart failure.
A. Shortness of breath
B. Orthopnea 20. You are assisting a patient up from the bed to
C. Edema the bathroom. The patient has swelling in the feet
D. Frothy-blood tinged sputum and legs. The patient is receiving treatment for
heart failure and is taking Hydralazine and Isordil.
16. These drugs are used as first-line treatment of Which of the following is a nursing priority for this
heart failure. They work by allowing more blood to patient while assisting them to the bathroom?
flow to the heart which decreases the work load of A. Measure and record the urine voided.
the heart and allows the kidneys to secrete sodium. B. Assist the patient up slowing and gradually.
However, some patients can develop a nagging C. Place the call light in the patient's reach while in
cough with these types of drugs. This description the bathroom.
describes? D. Provide privacy for the patient.
A. Beta-blockers
B. Vasodilators 21. A patient is taking Digoxin. Prior to
C. Angiotensin II receptor blockers administration you check the patient's apical pulse
D. Angiotensin-converting-enzyme inhibitors and find it to be 61 bpm. Morning lab values are the
following: K+ 3.3 and Digoxin level of 5 ng/mL.
17. A patient with heart failure is taking Losartan Which of the following is the correct nursing action?
and Spironolactone. The patient is having EKG A. Hold this dose and administer the second dose
changes that presents with tall peaked T-waves at 1800.
and flat p-waves. Which of the following lab results B. Administer the dose as ordered.
confirms these findings? C. Hold the dose and notify the physician of the
A. Na+ 135 digoxin level.
B. BNP 560 D. Hold this dose until the patient’s potassium level
C. K+ 8.0 is normal.
D. K+ 1.5
22. Which of the following is a common side effect
18. During your morning assessment of a patient of Spironolactone?
with heart failure, the patient complains of sudden A. Renal failure
vision changes that include seeing yellowish-green B. Hyperkalemia
halos around the lights. Which of the following C. Hypokalemia
medications do you suspect is causing this issue? D. Dry cough
A. Lisinopril
B. Losartan 23. The physician’s order says to administered
C. Lasix Lasix 40 mg IV twice a day. The patient has the
D. Digoxin following morning labs: Na+ 148, BNP 900, K+ 2.0,
and BUN 10. Which of the following is a nursing
priority?
A. Administer the Lasix as ordered
B. Notify the physician of the BNP level b. on the right side.
C. Assess the patient for edema c. in the tripod position.
D. Hold the dose and notify the physician about the d. in the high-Fowler's position.
potassium level
When admitting a patient in possible respiratory
1.3 Acute pulmonary failure failure with a high PaCO2, which assessment
To evaluate the effectiveness of prescribed information will be of most concern to the nurse?
therapies for a patient with ventilatory failure, which a. The patient is somnolent.
diagnostic test will be most useful to the nurse? b. The patient's SpO2 is 90%.
a. Chest x-rays c. The patient complains of weakness.
b. Pulse oximetry d. The patient's blood pressure is 162/94.
c. Arterial blood gas (ABG) analysis
d. Pulmonary artery pressure monitoring
A patient with acute respiratory distress syndrome
While caring for a patient who has been admitted (ARDS) and acute renal failure has the following
with a pulmonary embolism, the nurse notes a medications prescribed. Which medication should
change in the patient's oxygen saturation (SpO2) the nurse discuss with the health care provider
from 94% to 88%. The nurse will before administration?
a. increase the oxygen flow rate. a. ranitidine (Zantac) 50 mg IV
b. suction the patient's oropharynx. b. gentamicin (Garamycin) 60 mg IV
c. assist the patient to cough and deep breathe. c. sucralfate (Carafate) 1 g per nasogastric tube
d. help the patient to sit in a more upright position. d. methylprednisolone (Solu-Medrol) 40 mg IV
A patient with respiratory failure has a respiratory
rate of 8 and an SpO2 of 89%. The patient is A patient develops increasing dyspnea and
increasingly lethargic. The nurse will anticipate hypoxemia 2 days after having cardiac surgery. To
assisting with determine whether the patient has acute respiratory
a. administration of 100% oxygen by distress syndrome (ARDS) or pulmonary edema
non-rebreather mask. caused by left ventricular failure, the nurse will
b. endotracheal intubation and positive pressure anticipate assisting with
ventilation. a. inserting a pulmonary artery catheter.
c. insertion of a mini-tracheostomy with frequent b. obtaining a ventilation-perfusion scan.
suctioning. c. drawing blood for arterial blood gases.
d. initiation of bilevel positive pressure ventilation d. positioning the patient for a chest radiograph.
(BiPAP).
Which assessment finding by the nurse when
The pulse oximetry for a patient with right lower caring for a patient with ARDS who is being treated
lobe pneumonia indicates an oxygen saturation of with mechanical ventilation and high levels of
90%. The patient has rhonchi, a weak cough effort, positive end-expiratory pressure (PEEP) indicates
and complains of fatigue. Which action is best for that the PEEP may need to be decreased?
the nurse to take? a. The patient has subcutaneous emphysema.
a. Position the patient on the right side. b. The patient has a sinus bradycardia with a rate
b. Place a humidifier in the patient's room. of 52.
c. Assist the patient with staged coughing. c. The patient's PaO2 is 50 mm Hg and the SaO2 is
d. Schedule a 2-hour rest period for the patient. 88%.
d. The patient has bronchial breath sounds in both
When the nurse is caring for an obese patient with the lung fields.
left lower lobe pneumonia, gas exchange will be
best when the patient is positioned Which statement by the nurse when explaining the
a. on the left side. purpose of positive end-expiratory pressure (PEEP)
to the family members of a patient with ARDS is a. Encourage use of the incentive spirometer.
correct? b. Offer the patient fluids at frequent intervals.
a. "PEEP will prevent fibrosis of the lung from c. Teach the patient the importance of coughing.
occurring." d. Increase oxygen level to keep O2 saturation
b. "PEEP will push more air into the lungs during >95%.
inhalation."
c. "PEEP allows the ventilator to deliver 100% A patient with acute respiratory distress syndrome
oxygen to the lungs." (ARDS) who is intubated and receiving mechanical
d. "PEEP prevents the lung air sacs from collapsing ventilation develops a pneumothorax. Which action
during exhalation." will the nurse anticipate taking?
a. Lower the positive end-expiratory pressure
When prone positioning is used in the care of a (PEEP).
patient with acute respiratory distress syndrome b. Increase the fraction of inspired oxygen (FIO2).
(ARDS), which information obtained by the nurse c. Suction more frequently.
indicates that the positioning is effective? d. Increase the tidal volume.
a. The patient's PaO2 is 90 mm Hg, and the SaO2
is 92%. After receiving change-of-shift report, which patient
b. Endotracheal suctioning results in minimal will the nurse assess first?
mucous return. a. A patient with cystic fibrosis who has thick,
c. Sputum and blood cultures show no growth after green-colored sputum
24 hours. b. A patient with pneumonia who has coarse
d. The skin on the patient's back is intact and crackles in both lung bases
without redness. c. A patient with emphysema who has an oxygen
saturation of 91% to 92%
The nurse obtains the vital signs for a patient d. A patient with septicemia who has intercostal
admitted 2 days ago with gram-negative sepsis: and suprasternal retractions
temperature 101.2° F, blood pressure 90/56 mm
Hg, pulse 92, respirations 34. Which action should A patient with chronic obstructive pulmonary
the nurse take next? disease (COPD) arrives in the emergency
a. Administer the scheduled IV antibiotic. department complaining of shortness of breath and
b. Give the PRN acetaminophen (Tylenol) 650 mg. dyspnea. Which assessment finding by the nurse is
c. Obtain oxygen saturation using pulse oximetry. most important to report to the health care
d. Notify the health care provider of the patient's provider?
vital signs. a. The patient has bibasilar lung crackles.
b. The patient is sitting in the tripod position.
To decrease the risk for ventilator-associated c. The patient's respiratory rate has decreased from
pneumonia, which action will the nurse include in 30 to 10 breaths/min.
the plan of care for a patient who requires d. The patient's pulse oximetry indicates an O2
intubation and mechanical ventilation? saturation of 91%.
a. Avoid use of positive end-expiratory pressure
(PEEP). When assessing a patient with chronic lung
b. Suction every 2 hours. disease, the nurse finds a sudden onset of agitation
c. Elevate head of bed to 30 to 45 degrees. and confusion. Which action should the nurse take
d. Give enteral feedings at no more than 10 mL/hr. first?
a. Check pupil reaction to light.
A patient has a nursing diagnosis of ineffective b. Notify the health care provider.
airway clearance related to thick, secretions. Which c. Attempt to calm and reassure the patient.
action will be best for the nurse to include in the d. Assess oxygenation using pulse oximetry.
plan of care?
The nurse is caring for a 22-year-old patient who reabsorbed back into the bloodstream but is
came to the emergency department with acute excreted through the urine.
respiratory distress. Which information about the A. Urea
patient requires the most rapid action by the nurse? B. Creatinine
a. Respiratory rate is 32 breaths/min. C. Potassium
b. Pattern of breathing is shallow. D. Magnesium
c. The patient's PaO2 is 45 mm Hg.
d. The patient's PaCO2 is 34 mm Hg. 2. A patient with acute renal injury has a GFR
(glomerular filtration rate) of 40 mL/min. Which
The nurse is caring for a patient who was signs and symptoms below may this patient present
hospitalized 2 days earlier with aspiration with? Select all that apply:
pneumonia. Which assessment information is most A. Hypervolemia
important to communicate to the health care B. Hypokalemia
provider? C. Increased BUN level
a. Cough that is productive of blood-tinged sputum D. Decreased Creatinine level
b. Scattered crackles throughout the posterior lung
bases 3. You're assessing morning lab values on a female
c. Temperature of 101.5° F (38.6° C) after 2 days of patient who is recovering from a myocardial
IV antibiotic therapy infraction. Which lab value below requires you to
d. Oxygen saturation (SpO2) has dropped to 90% notify the physician?
with administration of 100% O2 by non-rebreather A. Potassium level 4.2 mEq/L
mask. B. Creatinine clearance 35 mL/min
C. BUN 20 mg/dL
Which of these nursing actions included in the care D. Blood pH 7.40
of a mechanically ventilated patient with acute
respiratory distress syndrome (ARDS) can the RN 4. A 55-year-old male patient is admitted with a
delegate to an experienced LPN/LVN working in the massive GI bleed. The patient is at risk for what
intensive care unit? type of acute kidney injury?
a. Assess breath sounds A. Post-renal
b. Insert a retention catheter B. Intra-renal
c. Place patient in the prone position C. Pre-renal
d. Monitor pulmonary artery pressures D. Intrinsic renal

A patient with ARDS who is receiving mechanical 5. Select all the patients below that are at risk for
ventilation using synchronized intermittent acute intra-renal injury?
mandatory ventilation (SIMV) has settings of A. A 45 year old male with a renal calculus.
fraction of inspired oxygen (FIO2) 80%, tidal B. A 65 year old male with benign prostatic
volume 500, rate 18, and positive end-expiratory hyperplasia.
pressure (PEEP) 5 cm. Which assessment finding C. A 25 year old female receiving chemotherapy.
is most important for the nurse to report to the D. A 36 year old female with renal artery stenosis.
health care provider? E. A 6 year old male with acute glomerulonephritis.
a. Oxygen saturation 99% F. An 87 year old male who is taking an
b. Patient respiratory rate 22 breaths/min aminoglycoside medication for an infection.
c. Crackles audible at lung bases
d. Apical pulse rate 104 beats/min 6. A patient with acute kidney injury has the
1.4 Acute Renal Failure following labs: GFR 92 mL/min, BUN 17 mg/dL,
1. ______________ is solely filtered from the potassium 4.9 mEq/L, and creatinine 1 mg/dL. The
bloodstream via the glomerulus and is NOT patient's 24 hour urinary output is 1.75 Liters.
Based on these findings, what stage of AKI is this 11. While educating a group of nursing students
patient in? about the stages of acute kidney injury, a student
A. Initiation asks how long the oliguric stage lasts. You explain
B. Diuresis to the student this stage can last?
C. Oliguric A. 1-2 weeks
D. Recovery B. 1-3 days
C. Few hours to 2 weeks
7. A 36-year-old male patient is diagnosed with D. 12 months
acute kidney injury. The patient is voiding 4 L/day of
urine. What complication can arise based on the 12. A patient with AKI has a urinary output of 350
stage of AKI this patient is in? Select all that apply: mL/day. In addition, morning labs showed an
A. Water intoxication increased BUN and creatinine level along with
B. Hypotension potassium level of 6 mEq/L. What type of diet
C. Low urine specific gravity ordered by the physician is most appropriate for
D. Hypokalemia this patient?
E. Normal GFR A. Low-sodium, high-protein, and low-potassium
B. High-protein, low-potassium, and low-sodium
8. True or False: All patients with acute renal injury C. Low-protein, low-potassium, and low-sodium
will progress through the oliguric stage of AKI but D. High-protein and high-potassium
not all patients will progress through the diuresis
stage.
True 1. While administering peritoneal dialysis to Mr.
False Thompson in the dialysis unit, Nurse Emily notes
that the return fluid is draining more slowly than
9. Which patient below with acute kidney injury is in usual. What is Nurse Emily's most appropriate
the oliguric stage of AKI: immediate course of action to address the slow
A. A 56 year old male who has metabolic acidosis, drainage of return fluid during Mr. Thompson's
decreased GFR, increased BUN/Creatinine, peritoneal dialysis?
hyperkalemia, edema, and urinary output 350 A. Elevate the drainage bag higher than Mr.
mL/day. Thompson's abdomen.
B. A 45 year old female with metabolic alkalosis, B. Inspect the outflow tubing for any kinks or
hypokalemia, normal GFR, increased obstructions.
BUN/creatinine, edema, and urinary output 600 C. Request Mr. Thompson to perform a cough.
mL/day. D. Adjust Mr. Thompson's bed to a reverse
C. A 39 year old male with metabolic acidosis, Trendelenburg position.
hyperkalemia, improving GFR, resolving edema,
and urinary output 4 L/day. 2. Nurse Emily is responsible for a patient who
D. A 78 year old female with respiratory acidosis, recently had an arteriovenous fistula placed for
increased GFR, decreased BUN/creatinine, hemodialysis. She knows it's crucial to regularly
hypokalemia, and urinary output 550 mL/day. assess for patency. What is the best method for
Emily to check the patency of this arteriovenous
10. You're developing a nursing care plan for a fistula?
patient in the diuresis stage of AKI. What nursing A. Evaluate capillary refill time in the nail beds of
diagnosis would you include in the care plan? the same arm.
A. Excess fluid volume B. Aspirate blood from the fistula using a needle
B. Risk for electrolyte imbalance and syringe.
C. Urinary retention C. Compress the fistula and observe the rate of
D. Acute pain refilling upon release.
D. Palpate along the entire length of the fistula for C. Overflow Incontinence.
the presence of a thrill. D. Stress Incontinence.

3. Nurse Sarah is caring for a paraplegic patient 7. Nurse James is crafting a care plan focused on
who has been diagnosed with renal calculi. Sarah the nursing diagnosis of "risk for infection" for his
is trying to understand the contributing factors that patient who has recently undergone a kidney
led to the formation of these calculi. Which factor is transplant. What would be an appropriate goal to
likely to have contributed to the development of set for this patient?
renal calculi in this patient? A. Return to regular employment within a period of
A. Enhanced loss of calcium from the skeletal 2 to 3 weeks post-surgery.
system. B. Maintain a normal body temperature and have
B. Lowered dietary calcium intake. cultures that show no infection.
C. Reduced renal function. C. Resume usual fluid consumption within a 48 to
D. Elevated fluid consumption. 72-hour timeframe.
D. Work toward ceasing the use of cyclosporine
4. Nurse Olivia is caring for a 32-year-old patient (Neoral) as expediently as safe.
and is reviewing the patient's medical chart. Based
on the symptoms exhibited, what would lead Nurse 8. Nurse Amanda is responsible for caring for Mr.
Olivia to suspect the patient may have acute Johnson, who has just undergone a prostatectomy.
glomerulonephritis? What complication requires immediate and priority
A. Symptoms of painful urination, frequent assessment in Mr. Johnson's post-operative care?
urination, and a sense of urgency. A. The onset of deep vein thrombosis symptoms.
B. Elevated blood pressure, reduced urine output, B. Early indications of pneumonia.
and complaints of fatigue. C. Signs of hemorrhaging or excessive bleeding.
C. Complaints of back pain accompanied by D. Symptoms of urine retention.
nausea and vomiting.
D. Symptoms of fever, chills, and pain in the right 9. Nurse Ethan is reviewing the medical histories of
upper quadrant radiating to the back. two different patients: one with renal failure and
another with prerenal failure. Ethan aims to
5. Nurse Olivia is caring for a patient who has differentiate the two conditions based on treatment
recently undergone a kidney transplant. She responsiveness. Which statement accurately
understands the importance of immunosuppressive distinguishes prerenal failure from renal failure?
therapy to prevent rejection of the new organ. For A. In prerenal failure, diuretics like furosemide
how long is immunosuppression generally (Lasix) are less effective in eliciting a response.
continued following a kidney transplantation? B. In prerenal failure, blood urea nitrogen (BUN)
A. Only for one week post-transplant. levels can be lowered through hemodialysis.
B. For the initial 24 hours following the transplant. C. In prerenal failure, vasoactive agents such as
C. Until there are no further signs of kidney dopamine (Intropin) elevate blood pressure levels.
rejection. D. In prerenal failure, administering an intravenous
D. Indefinitely, for the duration of the patient's life. infusion of isotonic saline enhances urine
production.
6. Nurse Ethan is reviewing the medical history of
Mrs. Clark, who reports that she leaks urine 10. Nurse Olivia is monitoring a postoperative
whenever she coughs, sneezes, or lifts heavy patient who has been back from surgery for six
items. Based on this description, what type of hours. The patient has an indwelling urinary
urinary incontinence is Mrs. Clark likely catheter, which was empty upon return but now
experiencing? shows only 120ml of urine. Olivia confirms that the
A. Urge Incontinence. drainage system is not obstructed. What is Olivia's
B. Reflex Incontinence. priority intervention in this situation?
A. Position the patient for shock management and What is the most frequently observed early sign of
alert the surgical team. kidney disease?
B. Assess the patient's circulatory status and vital A. Blood in the urine.
signs. B. Proteinuria.
C. Administer a 500ml bolus of isotonic saline C. Elevated blood pressure.
solution. D. Excessive thirst and hunger
D. Irrigate the urinary catheter with sterile saline or
water 15. Nurse Jane is closely monitoring a patient who
has been diagnosed with nephritic syndrome. Jane
11. Nurse Brian is reviewing the medical history of is eager to identify any positive indicators that
several patients admitted to the general medical would suggest her patient is on the path to
floor. Based on their profiles, which patient is at the recovery. What change would signify that the
greatest risk for developing a urinary tract infection patient with nephritic syndrome is recovering?
(UTI)? A. Decreased levels of serum albumin.
A. A 28-year-old individual diagnosed with angina. B. Complete absence of protein in the urine.
B. A 20-year-old woman diagnosed with asthma. C. Elevated levels of lipids in the blood serum.
C. A 50-year-old postmenopausal woman. D. An increase in overall body weight.
D. A 35-year-old woman who has a fractured wrist.
16. Nurse Laura is assessing a patient suspected
12. Nurse Olivia is caring for a patient who has of progressing through the stages of acute renal
diabetes mellitus and has recently started failure. What sign would indicate to her that the
hemodialysis due to renal failure. What would be patient is in the second phase of acute renal
the most appropriate dietary recommendation for failure?
this patient on the days between dialysis A. Stabilization of kidney function.
treatments? B. Urine production is less than 400 ml per day.
A. A low-protein diet with no restrictions on water C. A daily urine output that doubles, reaching 4 to 5
intake. liters per day.
B. No dietary restrictions whatsoever. D. A daily urine output of less than 100 ml.
C. A diet devoid of protein, along with the use of a 17. Nurse Sarah is conducting a follow-up visit in
salt substitute. the outpatient clinic for a patient who recently
D. A low-protein diet with a physician-prescribed received a kidney transplant. The patient reports
amount of water intake. discovering a lump in her breast. What should
Nurse Sarah consider regarding transplant
13. Nurse Anthony is caring for a patient diagnosed recipients?
with acute renal failure. He notes that the patient is A. More prone to tumor formation directly due to the
also experiencing elevated blood pressure levels. kidney transplant.
Anthony wants to identify the most prevalent cause B. At a lower likelihood for developing cancer, thus
of hypertension in the context of acute renal failure. the lump is probably non-malignant.
What is the most common cause of hypertension in C. Overwhelmed with anxiety post the life-altering
cases of acute renal failure? kidney transplant experience.
A. Excessive fluid volume in the body. D. At a heightened risk for cancer development
B. Fluid accumulation in the lungs. because of immunosuppressive therapy with
C. Low levels of red blood cells. cyclosporine (Neoral).
D. Insufficient fluid volume in the body.
18. Nurse Anthony is on his rounds when he
14. Nurse Olivia is conducting an educational checks on Mrs. Smith, who has recently been
session on the early signs of kidney disease for a diagnosed with acute pyelonephritis. Based on her
group of nursing students. She poses a question: diagnosis, which symptoms would Nurse Anthony
expect to observe in Mrs. Smith?
A. A burning sensation during urination. insulin schedule will prevent kidney damage. What
B. Jaundice and pain in the flank area. is the best response that Nurse Emily should give?
C. Increased frequency of urination and urination at A. "Your statement would hold true if diabetes could
night. solely be managed through insulin."
D. Tenderness at the costovertebral angle along B. "You should consult with your physician;
with chills. statistics suggest your viewpoint may be too
optimistic."
19. Nurse Karen is caring for a patient who has just C. "Despite following your insulin schedule, the risk
been wheeled back from the operating theater of kidney damage remains a concern."
following a repair of an abdominal aortic aneurysm. D. "As long as your kidneys continue to produce
She's vigilant for signs of potential complications, urine, you have minimal concern."
particularly acute renal failure. What symptom
should alert Nurse Karen to the possibility of acute 23. Nurse Emma is caring for a 60-year-old patient
renal failure in this postoperative patient? diagnosed with pyelonephritis and suspected
A. Frequent bowel movements. septicemia. The patient has had recurrent UTIs, is
B. Nausea and vomiting. sleep-deprived, and has lost weight due to frequent
C. Complete absence of urine. urination, including at night. The patient's lab
D. Oliguria results indicate sodium levels of 154 mEq/L,
20. Nurse Anna is evaluating a patient who may be osmolarity at 340 mOsm/L, glucose at 127 mg/dl,
suffering from a urinary tract infection (UTI). To and potassium at 3.9 mEq/L. What is the priority
further confirm the diagnosis, Anna pays close nursing diagnosis for this patient?
attention to the patient's subjective symptoms. A. Altered Nutrition: Less Than Body Requirements
Which statement from the patient would most likely related to the catabolic effects of insulin deficiency.
indicate the presence of a UTI? B. Altered Nutrition: Less Than Body Requirements
A. "I can go for long stretches without feeling the related to a hypermetabolic state.
need to urinate." C. Fluid Volume Deficit related to osmotic diuresis
B. "I've been urinating quite frequently." triggered by hyponatremia.
C. "I experience a burning sensation during D. Fluid Volume Deficit related to the inability to
urination." conserve water.
D. "There's a sweet odor coming from my urine."
24. Nurse Emily is attending to Lisa, a 30-year-old
21. Nurse David is overseeing the hemodialysis woman who is undergoing hemodialysis treatment.
treatment of a patient when he notices that she Lisa has an internal arteriovenous fistula in her arm
starts to become agitated. She complains of a for vascular access. Emily wants to take measures
headache and mentions feeling nauseous. David to minimize the risk of complications associated
needs to identify the likely complication that could with the arteriovenous fistula. What action should
be manifesting. What complication should Nurse Emily take to prevent complications related to this
David suspect based on the patient's symptoms vascular access device?
during hemodialysis? A. Establish intravenous lines proximal to the
A. Disequilibrium syndrome. location of the arteriovenous fistula.
B. Bacterial or viral infection. B. Refrain from measuring blood pressure on the
C. Entrapment of air in the bloodstream. arm containing the arteriovenous fistula.
D. Acute hemolysis. C. Feel for pulses distal to the arteriovenous fistula
site.
22. Nurse Emily is providing education to a D. Notify the healthcare provider if a bruit or thrill is
22-year-old patient with diabetic nephropathy. The detected over the arteriovenous fistula.
patient expresses the belief that since they are
young and have two kidneys, sticking to their 25. Nurse Lauren is caring for a patient, Mr. Patel,
who has been admitted with acute renal calculi and
is experiencing significant pain. What type of information should Nurse Zachary include in the
medication is generally indicated for the patient's medication teaching?
management of pain related to acute renal calculi? A. "You must complete this antibiotic course for a
A. Narcotic analgesics. total of 7 days."
B. Salicylates B. "Administer this medication between your meals
C. Nonsteroidal anti-inflammatory drugs (NSAIDs). and at bedtime."
D. Muscle relaxants. C. "Avoid this drug if you have a penicillin allergy."
D. "Expect your urine to possibly change to a bright
26. Nurse Melissa is caring for a patient with renal orange color."
failure and has orders to administer Polystyrene
sulfonate (Kayexalate). She understands that this 30. Nurse Emily is closely monitoring the cardiac
medication has a specific role in managing the status of a patient with end-stage renal disease.
complications associated with renal failure. What is The cardiac monitor starts showing frequent PVCs
the primary purpose of using Polystyrene sulfonate (Premature Ventricular Contractions). What is the
(Kayexalate) in patients with renal failure? priority nursing intervention for this situation?
A. To lower elevated serum phosphate A. Contact the healthcare provider immediately.
concentrations. B. Administer intravenous lidocaine (Xylocaine) to
B. To rectify acid-base imbalances. the patient.
C. To swap potassium ions for sodium ions. C. Review the most recent laboratory results for the
D. To counteract constipation resulting from sorbitol patient's potassium level.
usage. D. Ready the equipment for potential defibrillation
of the patient.
27. Nurse Jennifer is preparing discharge 31. Nurse John is formulating a care plan for a
instructions for a patient who was diagnosed with patient who is in end-stage renal disease. Among
sepsis stemming from a urinary tract infection the various aspects of patient care that need to be
(UTI). What key point should Nurse Jennifer include addressed, what would be the priority nursing
in her patient's discharge teaching plan? diagnosis for this patient?
A. Abstain from engaging in sexual activity. A. Reduced tolerance for physical activity.
B. Consume 8 to 10 eight-ounce glasses of water B. Excess fluid volume.
each day. C. Nutritional imbalance: insufficient dietary intake.
C. Refrain from using tampons. D. Elevated risk for physical harm.
D. Take cool baths to manage symptoms.
32. Nurse Alex is discussing dietary modifications
28. Nurse Laura is caring for Mr. Harris, who has with a patient who has both diabetes and a
been recently diagnosed with acute longstanding history of multiple renal stones. Now
glomerulonephritis. Based on the diagnosis, what facing chronic renal failure, which nutritional
clinical manifestations would Nurse Laura expect to component needs to be notably decreased in this
see in Mr. Harris? patient's meal plan?
A. Increased thirst and frequent urination. A. Carbohydrate intake
B. Reduced urine output and generalized swelling. B. Dietary fats
C. Chills and pain in the area between the ribs and C. Protein sources
hip. D. Ascorbic acid (Vitamin C)
D. Painful urination and low blood pressure.
33. Nurse William is speaking to a men's health
29. Nurse Zachary is preparing medication group about the importance of screening for
education for a patient diagnosed with a urinary prostate cancer. One of the attendees asks him
tract infection (UTI) and prescribed what is the most indicative diagnostic test for
phenazopyridine (Pyridium). What important detecting prostate cancer. What should Nurse
William reply?
A. The use of magnetic resonance imaging (MRI). C. Vigorously rub the affected skin areas with a
B. Excretory urography as a diagnostic measure. towel.
C. Testing for prostate-specific antigen (PSA) D. Maintain short, clean fingernails.
levels.
D. An in-depth digital rectal examination. 38. David, a dialysis nurse, is preparing a
54-year-old patient for hemodialysis. The patient
34. Nurse William is setting up for the urinary has a functioning arteriovenous (AV) fistula in
catheterization of a patient who has recently place. What should David prioritize to ensure the
suffered from a traumatic injury. As he gets ready, patency of the AV fistula before initiating
William notices that there's blood present at the hemodialysis?
opening of the urethra (urethral meatus). What A. Elevate the extremity with the AV fistula above
should be William's priority action in this situation? heart level.
A. Postpone the catheterization and alert the B. Apply a warm compress to the AV fistula site.
physician. C. Auscultate the AV fistula for bruit and palpate for
B. Analyze the discharge for hidden blood prior to thrill.
inserting the catheter. D. Administer a bolus of saline through the AV
C. Thoroughly rinse and clean the urethral opening fistula.
before proceeding with catheterization.
D. Generously apply lubrication to the catheter 39. Nurse Maya is working on a rehabilitation plan
before insertion. for a patient who has a neurogenic bladder. Which
action should be considered the most critical when
35. Nurse Anthony is taking care of a patient who undertaking bladder training for this patient?
has been diagnosed with end-stage renal disease. A. Suggest the continued use of an indwelling
What should be the priority nursing diagnosis for urinary catheter.
this patient? B. Promote the practice of Kegel exercises.
A. Intolerance to Physical Activity. C. Establish designated times for bladder emptying.
B. Excess Fluid Volume. D. Advocate for increased fluid intake.
C. Deficit in Patient Knowledge.
D. Experience of Pain. 40. Nurse Sarah is caring for a patient with
end-stage renal disease (ESRD) who has an
36. While attending to Mrs. Jones, who is currently arteriovenous fistula in the left arm for
in the oliguric phase of renal failure, Nurse hemodialysis. What intervention should be included
Rebecca wonders what level of 24-hour urine in the patient's plan of care?
output she should expect to be below for this A. Keep the left arm completely dry.
patient. B. Elevate the left arm on an arm board for a
A. Not exceeding 1000 milliliters. minimum of 30 minutes.
B. Less than 400 milliliters. C. Maintain the head of the bed at a 45-degree
C. Under 800 milliliters. elevation.
D. Below 200 milliliters. D. Apply consistent pressure to the needle insertion
site upon ceasing hemodialysis.
37. Nurse Sarah is reviewing the care plan for Mr.
Anderson, a patient with chronic renal failure who is 41. Nurse Laura is preparing a teaching care plan
complaining about persistent itching or pruritus. for a patient diagnosed with cystitis, who is being
What instruction should Nurse Sarah include in Mr. treated with phenazopyridine (Pyridium). Laura
Anderson's teaching plan to manage this wants to include essential guidance for the safe
symptom? and effective use of this medication. What
A. Take baths frequently to keep the skin clean. instruction should Nurse Laura include in the care
B. Utilize alcohol-based emollients for skin plan for this patient?
moisture.
A. Stop the course of prescribed antibiotics once A. Administer a 5% dextrose solution intravenously.
the painful urination is alleviated. B. Accelerate the rate of the ongoing hemodialysis
B. Contact the physician if your urine becomes treatment.
orange-red in color. C. Infuse a normal saline solution intravenously.
C. Administer phenazopyridine immediately prior to D. Urge Mrs. Wilson to perform active
urinating to alleviate discomfort. range-of-motion exercises.
D. Cease taking phenazopyridine as soon as
symptoms of painful urination subside 46. Nurse Megan is attending a continuing
education seminar on preventing urinary tract
42. Nurse David is developing a care plan for Ms. infections (UTIs) in hospitalized patients. The
Garcia, who has been diagnosed with renal calculi. speaker asks the audience which factor is primarily
What intervention should Nurse David include in responsible for the majority of UTIs in hospitalized
the care plan to best manage Ms. Garcia's settings. What is the most likely answer?
condition? A. The use of invasive procedures.
A. Encourage a diet high in purines. B. Inadequate fluid intake.
B. Limit fluid intake to reduce urine output. C. Immunosuppressed state of the patient.
C. Instruct the patient to strain all urine. D. Insufficient perineal hygiene.
D. Advise the patient to maintain complete bed rest.
47. Nurse Jake is monitoring a patient who has just
43. An 18-year-old college student is admitted to completed their first session of hemodialysis.
the hospital presenting with dark urine, fever, and Shortly afterward, the patient starts to experience a
flank pain. After diagnostic tests, the student is headache, elevated blood pressure, restlessness,
diagnosed with acute glomerulonephritis. What mental confusion, nausea, and vomiting. What
health history factor is most likely to be present in condition is most likely indicated by these
this student's case? symptoms?
A. Previous renal trauma. A. Disequilibrium Syndrome
B. Family history of acute glomerulonephritis. B. Respiratory Distress
C. History of renal calculi (kidney stones). C. Hypervolemia
D. A recent episode of a sore throat. D. Peritonitis

44. You're caring for a patient who recently 48. Nurse Isabella is working with a patient who
underwent a kidney transplant. You become may need to transition to continuous peritoneal
concerned about the possibility of kidney transplant dialysis. Before making any recommendations, she
rejection. Which symptoms would arouse your needs to evaluate whether the patient meets
suspicion? certain criteria to be eligible for this treatment
A. Discomfort at the surgical site, overall malaise, option. What is a required criterion for a patient to
and low blood pressure. be considered for continuous peritoneal dialysis?
B. Discomfort at the surgical site, overall feeling of A. Previous attempts at hemodialysis must have
weakness, and signs of depression. been unsuccessful.
C. Reduced urine production accompanied by low B. The patient must have severe pulmonary
blood pressure. edema.
D. Elevated body temperature, an increase in C. The patient's vascular access point needs to be
weight, and reduced urine production. fully healed.
D. The patient needs to have stable hemodynamic
45. During a session of hemodialysis, Nurse Alex parameters.
notices that her patient, Mrs. Wilson, starts to
complain about experiencing muscle cramps. What 49. Nurse Emily is assessing a patient who
should Nurse Alex do immediately to effectively presents with intense pain on the right side of his
relieve Mrs. Wilson's muscle cramps? lower back, nausea, vomiting, and restlessness.
The patient also appears somewhat pale and is Which assessment finding is commonly found in
sweating. His vital signs are: BP 140/90 mmHg, the oliguric phase of acute kidney injury (AKI)?
Pulse 118 beats/min., Respirations 33 breaths/min., A. Hypovolemia
and Temperature 98.0°F. Based on the subjective B. Hyperkalemia
data, which symptom most strongly suggests the C. Hypernatremia
diagnosis of renal calculi (kidney stones)? D. Thrombocytopenia
A. Vomiting that resembles coffee grounds in color.
B. Pain that extends toward the right upper Which patient has the greatest risk for prerenal
abdominal area. AKI?
C. Urine that is dark in color and produced in low A. The patient is hypovolemic because of
amounts. hemorrhage.
D. A history of experiencing mild symptoms of the B. The patient relates a history of chronic urinary
flu the previous week. tract obstruction.
C. The patient has vascular changes related to
50. Nurse Emily is caring for a patient who recently coagulopathies.
underwent surgery to create an arteriovenous D. The patient is receiving antibiotics such as
fistula for hemodialysis. What key information gentamicin.
should Nurse Emily keep in mind when providing
care for this patient? The patient admitted to the intensive care unit after
a motor vehicle accident has been diagnosed with
A. Auscultating the fistula with a stethoscope is not AKI. Which finding indicates the onset of oliguria
advised. resulting from AKI?
B. The patient should not experience pain during A. Urine output less than 1000 mL for the past 24
the initiation of dialysis. hours
C. Measuring blood pressure on the arm with the B. Urine output less than 800 mL for the past 24
arteriovenous fistula can lead to clot formation. hours
D. The patient generally feels at their best C. Urine output less than 600 mL for the past 24
immediately following dialysis treatment. hours
D. Urine output less than 400 mL for the past 24
hours
How do you determine that a patient's oliguria is
associated with acute renal failure (ARF)? The patient in the oliguric phase of AKI excreted
A. Specific gravity of urine at 3 different times is 300 mL of urine in addition to 100 mL of other
1.010. losses during the past 24 hours. With appropriate
B. The serum creatinine level is normal. calculations, you determine that for the next 24
C. The blood urea nitrogen (BUN) level is normal or hours the patient's fluid allocation is
below. A. 600 mL.
D. Hypokalemia is identified. B. 800 mL.
C. 1000 mL.
When caring for a patient during the oliguric phase D. 1200 mL.
of acute kidney injury, what would be an
appropriate nursing intervention? Your plan for care of a patient with AKI includes
A. Weigh patient three times weekly which goal of dietary management?
B. Increase dietary sodium and potassium A. Provide sufficient calories while preventing
C. Provide a low-protein, high-carbohydrate diet nitrogen excess.
D. Restrict fluids according to the previous day's B. Deliver adequate calories while restricting fat
fluid loss and protein intake.
C. Replace protein intake with enough fat intake to
sustain metabolism.
D. Restrict fluids, increase potassium intake, and C. Magnesium
regulate sodium intake. D. Phosphorus

For the patient with AKI, which laboratory result A patient is admitted to the hospital with CKD. You
would cause you the greatest concern? understand that this condition is characterized by
A. Potassium level of 5.9 mEq/L A. Progressive irreversible destruction of the
B. BUN level of 25 mg/dL kidneys
C. Sodium level of 144 mEq/L B. A rapid decrease in urinary output with an
D. pH of 7.5 elevated BUN level
C. Increasing creatinine clearance with a decrease
Important nursing interventions for the patient with in urinary output
AKI are (select all that apply) D. Prostration, somnolence, and confusion with
A. careful monitoring of intake and output. coma and imminent death
B. daily patient weights.
C. meticulous aseptic technique. Nurses need to educate patients at risk for CKD.
D. increase intake of vitamin A and D. Which individuals are considered to be at increased
E. frequent mouth care. risk (select all that apply)?
A. Older African Americans
What characterizes AKI (select all that apply)? B. Individuals older than 60 years
A. Primary cause of death is infection. C. Those with a history of pancreatitis
B. It usually affects older people. D. Those with a history of hypertension
C. The disease course is potentially reversible. E. Those with a history of type 2 diabetes
D. The most common cause is diabetic
nephropathy. Patients with CKD have an increased incidence of
E. Cardiovascular disease is the most common cardiovascular disease related to (select all that
cause of death. apply)
A. hypertension.
During the oliguric phase of AKI, you monitor the B. vascular calcifications.
patient for (select all that apply) C. a genetic predisposition.
A. hypertension. D. hyperinsulinemia causing dyslipidemia.
B. electrocardiographic (ECG) changes. E. increased high-density lipoproteins levels.
C. hypernatremia.
D. pulmonary edema. Measures indicated in the conservative therapy of
E. urine with high specific gravity. CKD include
A. decreased fluid intake, carbohydrate intake, and
protein intake.
If a patient is in the diuretic phase of AKI, you must B. increased fluid intake; decreased carbohydrate
monitor for which serum electrolyte imbalances? intake and protein intake.
A. Hyperkalemia and hyponatremia C. decreased fluid intake and protein intake;
B. Hyperkalemia and hypernatremia increased carbohydrate intake.
C. Hypokalemia and hyponatremia D. decreased fluid intake and carbohydrate intake;
D. Hypokalemia and hypernatremia increased protein intake.

You are preparing to administer a dose of PhosLo The advantage of continuous replacement therapy
to a patient with chronic kidney disease (CKD). This over hemodialysis is its ability to
medication should have a beneficial effect on which A. remove fluid without the use of a dialysate.
laboratory value? B. remove fluid in less than 24 hours.
A. Sodium C. allow the patient to receive the therapy at the
B. Potassium work site.
D. be administered through a peripheral line. B. The diet is less restricted and dialysis can be
performed at home.
You are caring for a patient receiving continuous C. The dialysate is biocompatible and causes no
replacement therapy and notice that the filtrate is long-term consequences.
blood tinged. What is your priority action? D. High glucose concentrations of the dialysate
A. Place the patient in Trendelenburg position. cause a reduction in appetite, promoting weight
B. Initiate a peripheral intravenous line. loss.
C. Suspend treatment immediately.
D. Administer vitamin K (Aquamephyton) per order. A patient is recovering in the intensive care unit
(ICU) after receiving a kidney transplant
A patient with a history of end-stage renal disease approximately 24 hours earlier. What is an
(ESRD) resulting from diabetes mellitus has expected assessment finding for this patient during
presented to the outpatient dialysis unit for his the early stage of recovery?
scheduled hemodialysis. Which assessment should A. Hypokalemia
you prioritize before, during, and after his B. Hyponatremia
treatment? C. Large urine output
A. Level of consciousness D. Leukocytosis with cloudy urine output
B. Blood pressure and fluid balance
C. Temperature, heart rate, and blood pressure An ESRD patient receiving hemodialysis is
D. Assessment for signs and symptoms of infection considering asking a relative to donate a kidney for
transplantation. In assisting the patient to make a
Which statement regarding continuous ambulatory decision about treatment, what do you tell the
peritoneal dialysis (CAPD) is of highest priority patient?
when teaching a patient new to this procedure? A. Successful transplantation usually provides
A. "It is essential that you maintain aseptic better quality of life than that offered by dialysis.
technique to prevent peritonitis." B. If rejection of the transplanted kidney occurs, no
B. "You will be allowed a more liberal protein diet further treatment for the renal failure is available.
after you complete CAPD." C. The immunosuppressive therapy that is required
C. "It is important for you to maintain a daily written after transplantation causes fatal malignancies in
record of blood pressure and weight." many patients.
D. "You must continue regular medical and nursing D. Hemodialysis replaces the normal functions of
follow-up visits while performing CAPD." the kidneys, and patients do not have to live with
the continual fear of rejection.
How should you assess the patency of a newly
placed arteriovenous graft for dialysis? A kidney transplant recipient complains of fever,
A. Irrigate the graft daily with low-dose heparin. chills, and dysuria over the past 2 days. What is the
B. Monitor for any increase in blood pressure in the first action that you should take?
affected arm. A. Assess temperature and initiate a workup to rule
C. Listen with a stethoscope over the graft for out infection.
presence of a bruit. B. Provide warm covers for the patient and give 1
D. Frequently monitor the pulses and gram of acetaminophen orally.
neurovascular status distal to the graft. C. Reassure the patient and let him know this is
common after transplantation.
What are the main advantages of peritoneal D. Notify the nephrologist that the patient has
dialysis compared to hemodialysis? developed symptoms of acute rejection.
A. No medications are required because of the
enhanced efficiency of the peritoneal membrane in
removing toxins. Which descriptions characterize AKI? Select all that
apply
a. primary cause of death is infection The patient has rapidly progressing glomerular
b. it almost always affects older people inflammation. Weight has increased and urine
c. disease course is potentially reversible output is steadily declining. What is the priority
d. most common cause is diabetic nephropathy nursing intervention?
e. cardiovascular disease is most common cause of a. Monitor the patient's cardiac status.
death b. Teach the patient about hand washing.
c. Obtain a serum specimen for electrolytes.
During the oliguric phase of AKI, the nurse monitors d. Increase direct observation of the patient.
the patient for Select all that apply
a. hypotension Which assessment findings would alert the nurse
b. ECG changes that the patient has entered the diuretic phase of
c. hypernatremia acute kidney injury (AKI)? Select all that apply
d. pulmonary edema a. Dehydration
e. urine with high specific gravity b. Hypokalemia
c. Hypernatremia
If a patient is in the diuretic phase of AKI, the nurse d. BUN increases
must monitor for which serum electrolyte e. Urine output increases
imbalances?
a. hyperkalemia and hyponatremia An unlicensed assistive personnel (UAP) reports to
b. hyperkalemia and hypernatremia the RN that a patient with acute kidney failure had a
c. hypokalemia and hyponatremia urine output of 350 mL over the past 24 hours after
d. hypokalemia and hypernatremia receiving furosemide 40 mg IV push. The UAP asks
the nurse how this can happen. What is the nurse's
The nurse is caring for a 68-yr-old man who had best response?
coronary artery bypass surgery 3 weeks ago.
During the oliguric phase of acute kidney disease, a. "During the oliguric phase of acute kidney failure,
which action would be appropriate to include in the patients often do not respond well to either fluid
plan of care? challenges or diuretics."
a. Provide foods high in potassium. b. "There must be some sort of error. Someone
b. Restrict fluids based on urine output. must have failed to record the urine output."
c. Monitor output from peritoneal dialysis. c. "A patient with acute kidney failure retains
d. Offer high-protein snacks between meals. sodium and water, which counteracts the action of
the furosemide."
When caring for a patient during the oliguric phase d. "The gradual accumulation of nitrogenous waste
of acute kidney injury (AKI), which nursing action is products results in the retention of water and
appropriate? sodium."
a. Weigh patient three times weekly.
b. Increase dietary sodium and potassium. The RN supervising a senior nursing student is
c. Provide a low-protein, high-carbohydrate diet. discussing methods for preventing acute kidney
d. Restrict fluids according to previous daily loss injury (AKI). Which points would the RN be sure to
include in this discussion? Select all that apply
Which patient diagnosis or treatment is most a. Encourage patients to avoid dehydration by
consistent with prerenal acute kidney injury (AKI)? drinking adequate fluids.
a. IV tobramycin b. Instruct patients to drink extra fluids during
b. Incompatible blood transfusion periods of strenuous exercise.
c. Poststreptococcal glomerulonephritis c. Immediately report a urine output of less than 2
d. Dissecting abdominal aortic aneurysm mL/kg/hr.
d. Record intake and output and weigh patients
daily.
e. Monitor laboratory values that reflect kidney
function. In a patient with acute kidney injury (AKI) who
requires hemodialysis, a temporary vascular
For which patient is the nurse most concerned access is obtained by placing a catheter in the left
about the risk for developing kidney disease? femoral vein. Which intervention will be included in
a. A 25-year-old patient who developed a urinary the plan of care?
tract infection (UTI) during pregnancy
b. A 55-year-old patient with a history of kidney a. Place the patient on bed rest.
stones b. Start continuous pulse oximetry.
c. A 63-year-old patient with type 2 diabetes c. Discontinue the retention catheter.
d. A 79-year-old patient with stress urinary d. Restrict the patients oral protein intake.
incontinence
Which information about a patient who was
A patient with acute kidney injury (AKI) has an admitted 10 days previously with acute kidney
arterial blood pH of 7.30. The nurse will assess the injury (AKI) caused by dehydration will be most
patient for important for the nurse to report to the health care
provider?
a. vasodilation. a. The blood urea nitrogen (BUN) level is 67 mg/dL.
b. poor skin turgor. b. The creatinine level is 3.0 mg/dL.
c. bounding pulses. c. Urine output over an 8-hour period is 2500 mL.
d. rapid respirations. d. The glomerular filtration rate is <30
mL/min/1.73m2.
A patient with severe heart failure develops
elevated blood urea nitrogen (BUN) and creatinine After noting lengthening QRS intervals in a patient
levels. The nurse will plan care to meet the goal of with acute kidney injury (AKI), which action should
a. replacing fluid volume. the nurse take first?
b. preventing hypertension. a. Document the QRS interval.
c. maintaining cardiac output. b. Notify the patients health care provider.
d. diluting nephrotoxic substances. c. Look at the patients current blood urea nitrogen
(BUN) and creatinine levels.
A patient who has acute glomerulonephritis is d. Check the chart for the most recent blood
hospitalized with acute kidney injury (AKI) and potassium level.
hyperkalemia. Which information will the nurse
obtain to evaluate the effectiveness of the When caring for a dehydrated patient with acute
prescribed calcium gluconate IV? kidney injury who is oliguric, anemic, and
a. Urine output hyperkalemic, which of the following prescribed
b. Calcium level actions should the nurse take first?
c. Cardiac rhythm a. Insert a urinary retention catheter.
d. Neurologic status b. Place the patient on a cardiac monitor.
c. Administer epoetin alfa (Epogen, Procrit).
Which information will be most useful to the nurse d. Give sodium polystyrene sulfonate (Kayexalate).
in evaluating improvement in kidney function for a
patient who is hospitalized with acute kidney injury What are intrarenal causes of AKI? Select all that
(AKI)? apply
a. anaphylaxis
a. Blood urea nitrogen (BUN) level b. renal stones
b. Urine output
c. Creatinine level c. nephrotoxic drugs
d. Calculated glomerular filtration rate (GFR) d. acute glomerulonephritis
e. tubular obstruction by myoglobin c. cardiac monitor reveals a depressed T wave and
elevated ST segment
An 83 year old female patient was found lying on d. the patient experiences increasing muscle
the bathroom floor. She said she fell 2 days ago weakness and abdominal cramping
and has not been able to take her heart medicine or
eat or drink anything since then. What conditions In caring for the patient with AKI, of what should the
could be causing prerenal AKI in this patient? nurse be aware?
Select all that apply a. the most common cause of death is irreversible
a. anaphylaxis metabolic acidosis
b. renal calculi b. during the oliguric phase, daily fluid intake is
c. hypovolemia limited to 1,000 ml plus the prior day's measured
d. nephrotoxic drugs fluid loss
e. decreased cardiac output c. dietary sodium and potassium during the oliguric
phase of AKI are managed according to the
ATN is the most common cause of intrarenal AKI. patient's urinary output
Which patient is most likely to develop ATN? d. one of the most important nursing measures in
a. patient with DM managing fluid balance in the patient with AKI is
b. patient with hypertensive crisis taking accurate daily weights
c. patient who tried to overdose on acetaminophen
d. patient with major surgery who required a blood A 68 year old man with a history of HF resulting
transfusion from HTN has AKI as a result of the effects of
nephrotoxic diuretics. Currently his serum
What indicates to the nurse that a patient with potassium is 6.2 with cardiac changes, BUN is 108,
oliguria has prerenal oliguria? serum creatinine 4.1, and serum HCO3 13. He is
a. urine testing reveals a low specific gravity somnolent and disoriented. Which treatment should
b. causative factor is malignant hypertension the nurse expect to be used for him?
c. urine testing reveals a high sodium concentration
d. reversal of oliguria occurs with fluid replacement a. loop diuretics
b. renal replacement therapy
Metabolic acidosis occurs in the oliguric phase of c. insulin and sodium bicarbonate
AKI as a result of impairment of d. sodium polystyrene sulfonate (kayexalate)
a. excretion of sodium
b. excretion of bicarbonate A patient with AKI has a serum potassium level of
c. conservation of potassium 6.7 and the following ABG results: pH: 7.28,
d. excretion of hydrogen ions PaCO2: 30, PaO2: 86, HCO3: 18. The nurse
recognizes that treatment of the acid-base problem
What indicates to the nurse that a patient with AKI with sodium bicarbonate would cause a decrease in
is in the recovery phase? which value?
a. a return to normal weight a. pH
b. a urine output of 3,700 mL/day b. potassium level
c. decreasing sodium and potassium levels c. bicarbonate level
d. decreasing BUN and creatinine levels d. carbon dioxide level

While caring for the patient in the oliguric phase of A patient with AKI is a candidate for continuous
AKI, the nurse monitors the patient for associated renal replacement therapy (CRRT). What is the
collaborative problems. When should the nurse most common indication for use of CRRT?
notify the HCP? a. pericarditis
a. urine output is 300 ml/day b. hyperkalemia
b. edema occurs in the feet, legs, and sacral area c. fluid overload
d. hypernatremia a. cardiac arrest
b. pulmonary edema
A nurse is planning care for a client who has c. circulatory collapse
prerenal AKI following abdominal aortic aneurysm d. hemorrhage
repair. Urinary output is 60 ml in the past 2 hours,
and BP is 92/58. The nurse should expect which of A high-carbohydrate, low-protein diet is prescribed
the following interventions? for the client with acute renal failure. The intended
a. prepare the client for a CT scan with contrast dye outcome of this diet is to
b. plan to administer nitroprusside a. act as a diuretic
c. prepare to administer a fluid challenge b. reduce demands on the liver
d. plan to position the client in Trendelenburg c. help maintain urine acidity
d. prevent the development of ketosis
A nurse is assessing a client who has prerenal AKI.
Which of the following findings should the nurse The client with acute renal failure asks the nurse for
expect? Select all that apply a snack. Because the client's potassium level is
a. reduced BUN elevated, which snack is most appropriate?
a. a gelatin dessert
b. elevated cardiac enzymes b. yogurt
c. reduced urine output c. an orange
d. elevated blood creatinine d. peanuts
e. elevated blood calcium
In the oliguric phase of acute renal failure, the
A client has been admitted with acute renal failure. nurse should assess the client for
What should the nurse do? Select all that apply a. pulmonary edema
a. elevate the HOB 30-45 degrees b. metabolic alkalosis
b. take vital signs c. hypotension
c. establish an IV site d. hypokalemia
d. call the admitting healthcare provider for
prescriptions The client in acute renal failure has an external
e. contact the hemodialysis unit cannula inserted in the forearm for hemodialysis.
Which nursing measure is appropriate for the care
Which initial manifestation of acute renal failure is of this client?
most common? a. use the unaffected arm for blood pressure
a. dysuria measurements
b. anuria b. draw blood from the cannula for routine
c. hematuria laboratory work
d. oliguria c. percuss the cannula for bruits each shift
d. inject heparin into the cannula each shift
The client who is in acute renal failure has an
elevated BUN. What is the likely cause of this During dialysis, the client has disequilibrium
finding? syndrome. The nurse should first
a. fluid retention a. administer oxygen per nasal cannula
b. hemolysis of RBCs b. slow the rate of dialysis
c. below-normal metabolic rate c. reassure the client that the symptoms are normal
d. reduced renal blood flow d. place the client in Trendelenburg's position

A client with acute renal failure has an increase in Which abnormal blood value would not be
the serum potassium level. The nurse should improved by dialysis treatment?
monitor the client for a. elevated serum creatinine level
b. hyperkalemia d. increase intake of vitamin A and D.
c. decreased hemoglobin concentration e. frequent mouth care.
d. hypernatremia

The client with acute renal failure is recovering and You expect a patient in the oliguric phase of renal
asks the nurse, "will my kidneys ever function failure to have a 24 hour urine output less than:
normally again?" The nurse's response is based on 1) 200ml
the knowledge that the client's renal status will most 2) 400ml
likely 3) 800ml
a. continue to improve over a period of weeks 4) 1000ml
b. result in the need for permanent hemodialysis
c. improve only if the client receives a renal The most common early sign of kidney disease is:
transplant 1. Sodium retention
d. result in end-stage renal failure 2. Elevated BUN level
3. Development of metabolic acidosis
A client with AKI has a serum potassium level of 4. Inability to dilute or concentrate urine
7.0. The nurse should plan which actions as a
priority? Select all that apply Which sign indicated the second phase of acute
a. place the client on a cardiac monitor renal failure?
b. notify the HCP 1. Daily doubling of urine output (4 to 5 L/day)
c. put the client on NPO status except for ice chips 2. Urine output less than 400 ml/day
d. review the client's medications to determine if 3. Urine output less than 100 ml/day
any contain or retain potassium 4. Stabilization of renal function
e. allow an extra 500 ml of IV fluid intake to dilute
the electrolyte concentration A patient who received a kidney transplant returns
for a follow-up visit to the outpatient clinic and
Which assessment finding is commonly found in reports a lump in her breast. Transplant recipients
the oliguric phase of acute kidney injury (AKI)? are:
a. Hypovolemia 1) At increased risk for cancer due to
b. Hyperkalemia immunosuppression caused by cyclosporine
c. Hypernatremia (Neoral)
d. Thrombocytopenia 2) Consumed with fear after the life-threatening
experience of having a transplant
Which patient has the greatest risk for prerenal 3) At increased risk for tumors because of the
AKI? kidney transplant
a. The patient who is hypovolemic because of 4) At decreased risk for cancer, so the lump is most
hemorrhage. likely benign
b. The patient who relates a history of chronic
urinary tract obstruction. You're developing a care plan with the nursing
c. The patient with vascular changes related to diagnosis risk for infection for your patient that
coagulopathies. received a kidney transplant. A goal for this patient
d. The patient receiving antibiotics such as is to:
gentamicin. 1) Remain afebrile and have negative cultures
2) Resume normal fluid intake within 2 to 3 days
Important nursing interventions for the patient with 3) Resume the patient's normal job within 2 to 3
AKI are Select all that apply weeks
a. careful monitoring of intake and output. 4) Try to discontinue cyclosporine (Neoral) as
b. daily patient weights. quickly as possible
c. meticulous aseptic technique.
You suspect kidney transplant rejection when the d) allow an extra 500 ml of fluid intake to dilute the
patient shows which symptoms? electrolyte concentration

1) Pain in the incision, general malaise, and


hypotension A adult client has had laboratory work done as part
2) Pain in the incision, general malaise, and of a routine physical examination. The nurse
depression interprets that the client may have a mild degree of
3) Fever, weight gain, and diminished urine output renal insufficiency if which of the following serum
4) Diminished urine output and hypotension creatinine levels is noted?

Which cause of hypertension is the most common a) 0.2 mg/dlL


in acute renal failure? b) 0.5 mg/dL
1) Pulmonary edema c) 1.9 mg/dL
2) Hypervolemia d) 3.5 mg/dL
3) Hypovolemia
4) Anemia The client in end-stage of renal failure had
undergone kidney transplant. Which of the following
Which statement correctly distinguishes renal assessment findings indicate kidney transplant
failure from prerenal failure? rejection?

1) With prerenal failure, vasoactive substances a) increased urinary output, BUN = 15 mg/dL
such as dopamine (Intropin) increase blood b) HCT = 50%, Hgb = 17 g/dl
pressure c) decreased urinary output, sudden weight gain
2) With prerenal failure, there is less response to d) decreased urinary output, sudden weight loss
such diuretics as furosemide (Lasix)
3) With prerenal failure, an IV isotonic saline Which of the following anti-hypertensive
infusion increases urine output medications is contraindicated for clients with renal
4) With prerenal failure, hemodialysis reduces the insufficiency?
BUN level a) beta-adrenergic blockers
b) calcium-channel blockers
The home care nurse is making follow-up visits to a c) direct-acting vasodilators
client following renal transplant. The nurse d) angiotensin-converting enzyme inhibitors
assesses the client for which signs of acute graft
rejection? In the oliguric phase of renal failure, what is the
most appropriate nursing diagnosis?
a) hypotension, graft tenderness, and anemia a) fluid volume deficit
b) hypertension, oliguria, thirst, and hypothermia b) activity intolerance
c) fever, hypertension, graft tenderness, and c) ineffective breathing pattern
malaise d) fluid volume excess
d) fever, vomiting, hypotension, and copious
amounts of dilute urine A patient with acute renal failure (ARF) has an
arterial blood pH of 7.30. The nurse will assess the
The client with acute renal failure has a serum patient for
potassium of 6.0 mEq/L. The nurse would plan a. tachycardia.
which of the following as a priority action? b. rapid respirations.
a) check the sodium level c. poor skin turgor.
b) place the client on a cardiac monitor d. vasodilation.
c) encourage increased vegetables in the diet
A patient with severe heart failure develops d. draw blood to monitor for hyperkalemia.
elevated BUN and creatinine levels. The nurse
plans care for the patient based on the knowledge After noting increasing QRS intervals in a patient
that collaborative care of the patient will be directed with ARF, which action should the nurse take first?
toward the goal of a. Notify the patient's health care provider.
a. preventing hypertension. b. Check the chart for the most recent blood
b. replacing fluid volume. potassium level.
c. diluting nephrotoxic substances. c. Look at the patient's current BUN and creatinine
d. maintaining cardiac output. levels.
d. Document the QRS interval.
A patient admitted with sepsis has had several
episodes of severe hypotension. Laboratory results A patient with renal insufficiency is scheduled for an
indicate a BUN 50 mg/dl (10.7 mmol/L), serum intravenous pyelogram (IVP). Which of the
creatinine 2.0 mg/dl (177 µmol/L), urine sodium 70 following orders for the patient will the nurse
mEq/L (70 mmol/L), urine specific gravity 1.010, question?
and cellular casts and debris in the urine. The a. Ibuprofen (Advil) 400 mg PO PRN for pain
nurse knows these findings are consistent with b. Dulcolax suppository 4 hours before IVP
a. chronic renal insufficiency. procedure
b. prerenal failure. c. Normal saline 500 ml IV before procedure
c. postrenal failure. d. NPO for 6 hours before IVP procedure
d. acute tubular necrosis.
Before administering sodium polystyrene sulfonate
A patient in the oliguric phase of acute renal failure (Kayexalate) to a patient with hyperkalemia, the
has a 24-hour fluid output of 150 ml emesis and nurse should assess
250 ml urine. The nurse plans a fluid replacement a. the BUN and creatinine.
for the following day of ___ ml. b. the blood glucose level.
a. 400 c. the patient's bowel sounds.
b. 800 d. the level of consciousness (LOC).
c. 1000
d. 1400 In the immediate postoperative period, the nurse
caring for a patient who is a recipient of a kidney
The health care provider orders IV glucose and transplant would expect that fluid therapy would
insulin to be given to a patient in ARF whose serum involve administration of IV fluids
potassium level is 6.3 mEq/L. To best evaluate the a. to be determined hourly, based on every milliliter
effectiveness of the medications, the nurse will of urine output.
a. monitor the patient's electrocardiograph (ECG). b. at a minimum rate of 100 ml/hr to perfuse the
b. check the blood glucose level. kidney.
c. obtain serum potassium levels. c. titrated to keep blood pressure within a normal
d. assess BUN and creatinine levels. range.
d. at a rate to keep urine clear and without blood
clots.
A patient in ARF has a gradual increase in urinary
output to 3400 ml a day with a BUN of 92 mg/dl (33 To monitor for corticosteroid-related complications
mmol/L) and a serum creatinine of 4.2 mg (371 after a kidney transplant, the nurse teaches the
μmol/L). The nurse should plan to patient to report
a. use a urine dipstick to monitor for proteinuria. a. pain at the donor kidney site.
b. auscultate the lungs to assess for pulmonary b. dizziness with position change.
edema. c. pain in the hips, knees, and other joints.
c. take the blood pressure to check for hypotension. d. changes in the character of the urine.
4) Client's support system and understanding of
Two hours after a kidney transplant, the nurse lifestyle changes.
obtains the following pieces of data when
assessing the patient. Which information is most Which of the following symptoms indicate acute
important to communicate to the health care rejection of a transplanted kidney?
provider? 1) Edema, nausea
a. The BUN and creatinine levels are elevated. 2) Fever, anorexia
b. The urine output is 900 to 1100 ml/hr. 3) Weight gain, pain at graft site
c. The patient's central venous pressure (CVP) is 4) Increased WBC count, pain with voiding
decreased.
d. The patient has level 8 (on a 10-point scale) Adverse reactions of prednisone therapy include
incision pain when coughing. which of the following conditions?
1) Acne and bleeding gums
Which data obtained when assessing a patient who 2) Sodium retention and constipation
had a kidney transplant 8 years ago and who is 3) Mood swings and increased temperature
receiving the immunosuppressants tacrolimus 4) Increased blood glucose levels and decreased
(Prograf), cyclosporine (Sandimmune), and wound healing.
prednisone (Deltasone) will be of most concern to
the nurse? The nurse is caring for a client following a kidney
a. The blood glucose is 144 mg/dl. transplant. The client develops oliguria. Which of
b. The patient has a round, moonlike face. the following would the nurse anticipate to be
c. There is a nontender lump in the axilla. prescribed as the treatment of oliguria?
d. The patient's blood pressure is 150/92. 1) Encourage fluid intake
2) Administration of diuretics
A client has just received a renal transplant and has 3) Irrigation of foley catheter
started cyclosporine therapy to prevent graft 4) Restricting fluids
rejection. Which of the following conditions is a
major complication of this drug therapy? A hospitalized patient is in the oliguric phase of
1) Depression acute renal failure. The nurse should implement
2) Hemorrhage which of the following?
3) Infection Select all that apply.
4) Peptic ulcer disease A. Keep strict intake and output records.
B. Closely monitor potassium levels.
A client received a kidney transplant 2 months ago. C. Administer dopamine to increase renal
He's admitted to the hospital with the diagnosis of perfusion.
acute rejection. Which of the following assessment D. Limit fluid intake.
findings would be expected? E. Administer laxatives to prevent fluid overload.
1) Hypotension
2) Normal body temperature A patient has developed acute renal failure (ARF)
3) Decreased WBC count with a 24-hour urine output of 350 mL. The nurse
4) Elevated BUN and creatinine levels should assess the patient for:
Pruritus
The client is to undergo kidney transplantation with Polyphagia
a living donor. Which of the following preoperative Cardiac arrhythmias
assessments is important? Tetany
1) Urine output
2) Signs of graft rejection For a male client in the oliguric phase of acute renal
3) Signs and symptoms of rejection failure (ARF), which nursing intervention is most
important?
A. Encouraging coughing and deep breathing D. Ischemic stenosis
B. Promoting carbohydrate intake
C. Limiting fluid intake 2. Which patient below is at most risk for a
D. Providing pain-relief measures hemorrhagic stroke?
A. A 65 year old male patient with carotid stenosis.
B. A 89 year old female with atherosclerosis.
The nurse is reviewing laboratory results on a client C. A 88 year old male with uncontrolled
with acute renal failure. Which one of the following hypertension and a history of brain aneurysm repair
should be reported IMMEDIATELY? 2 years ago.
a. Blood urea nitrogen 50 mg/dl D. A 55 year old female with atrial flutter.
b. Hemoglobin of 10.3 mg/dl
c. Venous blood pH 7.30 3. You're educating a patient about transient
d. Serum potassium 6 mEq/L ischemic attacks (TIAs). Select all the options that
are incorrect about this condition:
A client with acute renal failure moves into the A. TIAs are caused by a temporary decrease in
diuretic phase after 1 week of therapy. During this blood flow to the brain.
phase, the client must be assessed for signs of B. TIAs produce signs and symptoms that can last
developing: for several weeks to months.
a. Renal failure C. A TIAs is a warning sign that an impending
b. Hypovolemia stroke may occur.
c. Hyperkalemia D. TIAs don't require medical treatment.
d. Metabolic acidosis
4. A patient who suffered a stroke one month ago is
A male client develops acute renal failure (ARF) experiencing hearing problems along with issues
after receiving I.V. therapy with a nephrotoxic learning and showing emotion. On the MRI what
antibiotic. Because the client's 24-hour urine output lobe in the brain do you expect to be affected?
totals 240 ml, nurse Andy suspects that the client is A. Frontal lobe
at risk for: B. Occipital lobe
a. cardiac arrhythmia. C. Parietal lobe
b. paresthesia. D. Temporal
c. dehydration.
d. pruritus. 5. A patient's MRI imaging shows damage to the
cerebellum a week after the patient suffered a
A hospitalized patient is in the oliguric phase of stroke. What assessment findings would correlate
acute renal failure. The nurse should implement with this MRI finding?
which of the following interventions? A. Vision problems
A. Administer laxatives to prevent fluid overload B. Balance impairment
B. Limit fluid intake C. Language difficulty
C. Closely monitor hemoglobin level D. Impaired short-term memory
D. Administer dopamine to increase renal perfusion
6. A patient is demonstrating signs and symptoms
1.5 Stroke of stroke. The patient reports loss of vision. What
1. A patient is admitted with uncontrolled atrial area of the brain do you suspect is affected based
fibrillation. The patient’s medication history includes on this finding?
vitamin D supplements and calcium. What type of A. Brain stem
stroke is this patient at MOST risk for? B. Hippocampus
A. Ischemic thrombosis C. Parietal lobe
B. Ischemic embolism D. Occipital lobe
C. Hemorrhagic
7. A patient has right side brain damage from a B. Don't repeat questions.
stroke. Select all the signs and symptoms that C. Ask questions that require a simple response.
occur with this type of stroke: D. Use a communication board.
A. Right side hemiplegia E. Discourage the patient from using words.
B. Confusion on date, time, and place
C. Aphasia 12. While conversing with a patient who had a
D. Unilateral neglect stroke six months ago, you note their speech is
E. Aware of limitations hard to understand and slurred. This is known as:
F. Impulsive A. Dysarthria
G. Short attention span B. Apraxia
H. Agraphia C. Alexia
D. Dysphagia
8. You're educating a group of nursing students
about left side brain damage. Select all the signs 13. You're reading the physician’s history and
and symptoms noted with this type of stroke: physical assessment report. You note the physician
A. Aphasia wrote that the patient has apraxia. What
B. Denial about limitations assessment finding in your morning assessment
C. Impaired math skills correlates with this condition?
D. Issues with seeing on the right side A. The patient is unable to read.
E. Disoriented B. The patient has limited vision in half of the visual
F. Depression and anger field.
G. Impulsive C. The patient is unable to wink or move his arm to
H. Agraphia scratch his skin.
D. The patient doesn't recognize a pencil or
television.
9. During discharge teaching for a patient who
experienced a mild stroke, you are providing details 14. You need to obtain informed consent from a
on how to eliminate risk factors for experiencing patient for a procedure. The patient experienced a
another stroke. Which risk factors below for stroke stroke three months ago. The patient is unable to
are modifiable? sign the consent form because he can't write. This
A. Smoking is known as what:
B. Family history A. Agraphia
C. Advanced age B. Alexia
D. Obesity C. Hemianopia
E. Sedentary lifestyle D. Apraxia

10. Your patient who had a stroke has issues with 15. You're assessing your patient's pupil size and
understanding speech. What type of aphasia is this vision after a stroke. The patient says they can only
patient experiencing and what area of the brain is see half of the objects in the room. You document
affected? this finding as:
A. Expressive; Wernicke's area A. Hemianopia
B. Receptive, Broca's area B. Opticopsia
C. Expressive; hippocampus C. Alexia
D. Receptive; Wernicke's area D. Dysoptic

11. Your patient has expressive aphasia. Select all 16. A patient who has hemianopia is at risk for
the ways to effectively communicate with this injury. What can you educate the patient to perform
patient? regularly to prevent injury?
A. Fill in the words for the patient they can't say. A. Wearing anti-embolism stockings daily
B. Consume soft foods and tuck in chin while C. Ask direct questions that require one word
swallowing responses.
C. Scanning the room from side to side frequently D. Offer the bedpan and bedside commode every 2
D. Muscle training hours.

17. You receive a patient who is suspected of The nurse is caring for a patient with a history of
experiencing a stroke from EMS. You conduct a transient ischemic attacks (TIAs) and moderate
stroke assessment with the NIH Stroke Scale. The carotid stenosis who has undergone a carotid
patient scores a 40. According to the scale, the endarterectomy. Which of the following
result is: postoperative findings would cause the nurse the
A. No stroke symptoms most concern?
B. Severe stroke symptoms a) Blood pressure (BP): 128/86 mm Hg
C. Mild stroke symptoms b) Neck pain: 3/10 (0 to 10 pain scale)
D. Moderate stroke symptoms c) Mild neck edema
d) Difficulty swallowing
18. In order for tissue plasminogen activator (tPA)
to be most effective in the treatment of stroke, it An emergency department nurse is interviewing a
must be administered? client with signs of an ischemic stroke that began 2
A. 6 hours after the onset of stroke symptoms hours ago. The client reports that she had a
B. 3 hours before the onset of stroke symptoms cholecystectomy 6 weeks ago and is taking digoxin,
C. 3 hours after the onset of stroke symptoms coumadin, and labetelol. This client is not eligible
D. 12 hours before the onset of stroke symptoms for thrombolytic therapy for which of the following
reasons?
19. Which patients are NOT a candidate for tissue a) She is not within the treatment time window.
plasminogen activator (tPA) for the treatment of b) She had surgery 6 weeks ago.
stroke? c) She is taking digoxin.
A. A patient with a CT scan that is negative. d) She is taking coumadin.
B. A patient whose blood pressure is 200/110.
C. A patient who is showing signs and symptoms of Which disturbance results in loss of half of the
ischemic stroke. visual field?
D. A patient who received Heparin 24 hours ago. a) Anisocoria
b) Homonymous hemianopsia
20. You're assisting a patient who has right side c) Nystagmus
hemiparesis and dysphagia with eating. It is very d) Diplopia
important to:
A. Keep the head of bed less than 30'. A client with a cerebrovascular accident (CVA) is
B. Check for pouching of food in the right cheek. having difficulty with eating food on the plate.
C. Prevent aspiration by thinning the liquids. Which is the best nursing action to be taken?
D. Have the patient extend the neck upward away a) Reposition the tray and plate.
from the chest while eating. b) Perform a vision field assessment.
c) Know this is a normal finding for CVA.
21. A patient has experienced right side brain d) Assist the client with feeding.
damage. You note the patient is experiencing
neglect syndrome. What nursing intervention will A client is hospitalized when they present to the
you include in the patient's plan of care? Emergency Department with right-sided weakness.
A. Remind the patient to use and touch both sides Within 6 hours of being admitted, the neurologic
of the body daily. deficits had resolved and the client was back to
B. Offer the patient a soft mechanical diet with their presymptomatic state. The nurse caring for the
honey thick liquids.
client knows that the probable cause of the Which of the following insults or abnormalities can
neurologic deficit was what? cause an ischemic stroke?
a) Cerebral aneurysm a) Arteriovenous malformation
b) Transient ischemic attack b) Intracerebral aneurysm rupture
c) Left-sided stroke c) Cocaine use
d) Right-sided stroke d) Trauma

Which of the following terms refer to the failure to A nurse is working with a student nurse who is
recognize familiar objects perceived by the senses? caring for a client with an acute bleeding cerebral
a) Agnosia aneurysm. Which action by the student nurse
b) Perseveration requires further intervention?
c) Apraxia a) Maintaining the client in a quiet environment
d) Agraphia b) Keeping the client in one position to decrease
bleeding
During a class on stroke, a junior nursing student
asks what the clinical manifestations of stroke are. c) Positioning the client to prevent airway
What would be the instructor's best answer? obstruction
a) "Clinical manifestations of a stroke depend on d) Administering I.V. fluid as ordered and
the area of the cortex, the affected hemisphere, the monitoring the client for signs of fluid volume
degree of blockage, and the availability of collateral excess
circulation."
b) "Clinical manifestations of a stroke generally The nurse is caring for a patient with aphasia.
include aphasia, one-sided flaccidity, and trouble Which of the following strategies will the nurse use
swallowing." to facilitate communication with the patient?
a) Speaking in complete sentences
c) "Clinical manifestations of a stroke depend on b) Speaking loudly
how quickly the clot can be dissolved." c) Avoiding the use of hand gestures
d) "Clinical manifestations of a stroke are highly d) Establishing eye contact
variable, depending on the cardiovascular health of
the client." While providing information to a community group,
the nurse tells them the primary initial symptoms of
When communicating with a client who has sensory a hemorrhagic stroke are:
(receptive) aphasia, the nurse should: a) Footdrop and external hip rotation
a) speak loudly and articulate clearly. b) Severe headache and early change in level of
b) allow time for the client to respond. consciousness
c) give the client a writing pad. c) Weakness on one side of the body and difficulty
d) use short, simple sentences. with speech
d) Confusion or change in mental status
The nurse is providing information about strokes to
a community group. Which of the following would A client is admitted with weakness, expressive
the nurse identify as the primary initial symptoms of aphasia, and right hemianopia. The brain MRI
an ischemic stroke? reveals an infarct. The nurse understands these
a) Footdrop and external hip rotation symptoms to be suggestive of which of the
b) Vomiting and seizures following findings?
c) Severe headache and early change in level of a) Left-sided cerebrovascular accident (CVA)
consciousness b) Right-sided cerebrovascular accident (CVA)
d) Weakness on one side of the body and difficulty c) Transient ischemic attack (TIA)
with speech d) Completed Stroke
A nurse is caring for a client who has returned to The provider diagnoses the patient as having had
his room after a carotid endarterectomy. Which an ischemic stroke. The etiology of an ischemic
action should the nurse take first? stroke would include which of the following?
a) Cerebral aneurysm
b) Cardiogenic emboli
a) Take the client's blood pressure. c) Intracerebral hemorrhage
b) Ask the client if he has a headache. d) Arteriovenous malformation
c) Ask the client if he has trouble breathing.
d) Place antiembolism stockings on the client. A patient is in the acute phase of an ischemic
stroke. How long does the nurse know that this
The nurse is caring for a patient with dysphagia. phase may last?
Which of the following interventions would be a) Up to 1 week
contraindicated while caring for this patient? b) Up to 24 hours
a) Allowing ample time to eat c) 1 to 3 days
b) Assisting the patient with meals d) Up to 2 weeks
c) Testing the gag reflex prior to offering food or
fluids Which of the following terms refer to the inability to
d) Placing food on the affected side of mouth perform previously learned purposeful motor acts
on a voluntary basis?
A 64-year-old client reports symptoms consistent a) Agraphia
with a transient ischemic attack (TIA) to the b) Perseveration
physician in the emergency department. After c) Agnosia
completing ordered diagnostic tests, the physician d) Apraxia
indicates to the client what caused the symptoms
that brought him to the hospital. What is the origin A nurse is working with a student nurse who is
of the client's symptoms? caring for a client with an acute bleeding cerebral
a) Hypertension aneurysm. Which action by the student nurse
b) Cardiac disease requires further intervention?
c) Diabetes insipidus a) Maintaining the client in a quiet environment
d) Impaired cerebral circulation b) Positioning the client to prevent airway
obstruction
A client is admitted to the intensive care unit (ICU)
with a diagnosis of cerebrovascular accident (CVA). c) Keeping the client in one position to decrease
Which assessment by the nurse provides the most bleeding
significant finding in differentiating between d) Administering I.V. fluid as ordered and
ischemic and hemorrhagic strokes? monitoring the client for signs of fluid volume
a) Oropharyngeal suctioning as needed. excess
b) Kepprais ordered for treatment of focal seizures.
c) A unit of fresh frozen plasma is infusing. A client has experienced an ischemic stroke that
d) Neurological checks are ordered every 2 hours. has damaged the lower motor neurons of the brain.
Which of the following deficits would the nurse
The nurse is caring for a patient with aphasia. expect during assessment?
Which of the following strategies will the nurse use a) Limited attention span and forgetfulness
to facilitate communication with the patient? b) Visual agnosia
a) Avoiding the use of hand gestures c) Auditory agnosia
b) Establishing eye contact d) Lack of deep tendon reflexes
c) Speaking in complete sentences
d) Speaking loudly
A nurse knows that, for a patient with an ischemic b) Obesity
stroke, tPA is contraindicated if the blood pressure c) Hypertension
reading is: d) Smoking
a) 170 mm Hg/105 mm Hg
b) 185 mm Hg/110 mm Hg While providing information to a community group,
c) 190 mm Hg/120 mm Hg the nurse tells them the primary initial symptoms of
d) 175 mm Hg/100 mm Hg a hemorrhagic stroke are:
a) Footdrop and external hip rotation
A patient who has suffered a stroke begins having b) Severe headache and early change in level of
complications regarding spasticity in the lower consciousness
extremity. What ordered medication does the nurse c) Confusion or change in mental status
administer to help alleviate this problem? d) Weakness on one side of the body and difficulty
a) Pregabalin (Lyrica) with speech
b) Diphenhydramine (Benadryl)
c) Heparin A client is admitted with weakness, expressive
d) Lioresal (Baclofen) aphasia, and right hemianopia. The brain MRI
reveals an infarct. The nurse understands these
A patient diagnosed with a stroke is ordered to symptoms to be suggestive of which of the
receive warfarin (Coumadin). Later, the nurse following findings?
learns that the warfarin is contraindicated and the a) Left-sided cerebrovascular accident (CVA)
order is canceled. The nurse knows that the best b) Completed Stroke
alternative medication to give is which of the c) Transient ischemic attack (TIA)
following? d) Right-sided cerebrovascular accident (CVA)
a) Ticlodipine (Ticlid)
b) Dipyridamole (Persantine) A physician orders several drugs for a client with
c) Clopidogrel (Plavix) hemorrhagic stroke. Which drug order should the
d) Aspirin nurse question?
a) Phenytoin (Dilantin)
Which of the following is the most common side b) Methyldopa (Aldomet)
effect of tissue plasminogen activator (tPA)? c) Heparin sodium
a) Increased intracranial pressure (ICP) d) Dexamethasone (Decadron)
b) Hypertension
c) Headache A patient presents to the emergency room with
d) Bleeding complaints of having an "exploding headache" for
the last 2 hours. The patient is immediately seen by
Which of the following antiseizure medication has a triage nurse who suspects the patient is
been found to be effective for post-stroke pain? experiencing a stroke. Which of the following is a
a) Carbamazepine (Tegretol) possible cause based on the characteristic
b) Lamotrigine (Lamictal) symptom?
c) Topiramate (Topamax) a) Cerebral aneurysm
d) Phenytoin (Dilantin) b) Cardiogenic emboli
c) Large artery thrombosis
The nurse practitioner advises a patient who is at d) Small artery thrombosis
high risk for a stroke to be vigilant in his medication
regime, to maintain a healthy weight, and to adopt Which of the following is accurate regarding a
a reasonable exercise program. This advice is hemorrhagic stroke?
based on research data that shows the most a) It is caused by a large-artery thrombosis.
important risk factor for stroke is: b) One of the main presenting symptoms is
a) Dyslipidemia numbness or weakness of the face.
c) Main presenting symptom is an "exploding confirmed and the doctor ordered tPA. The nurse
headache." knows to give this drug no later than what time?
d) Functional recovery usually plateaus at 6 a) 5:30 p.m.
months. b) 3:00 p.m.
c) 4:00 p.m.
A 73-year-old client is visiting the neurologist. The d) 2:30 p.m.
client reports light-headedness, speech
disturbance, and left-sided weakness that have The nurse is providing diet-related advice to a male
lasted for several hours. In the examination, an patient following a cerebrovascular accident (CVA).
abnormal sound is auscultated in an artery leading The patient wants to minimize the volume of food
to the brain. What is the term for the auscultated and yet meet all nutritional elements. Which of the
discovery? following suggestions should the nurse give to the
a) Atherosclerotic plaque patient about controlling the volume of food intake?
b) TIA a) Provide a high-fat diet.
c) Diplopia b) Include dry or crisp foods and chewy meats.
d) Bruit c) Always serve hot or tepid foods.
d) Provide thickened commercial beverages and
Which of the following statements reflect nursing fortified cooked cereals.
management of the patient with expressive
aphasia? A client is receiving an I.V. infusion of mannitol
a) Frequently reorient the patient to time, place, (Osmitrol) after undergoing intracranial surgery to
and situation remove a brain tumor. To determine whether this
b) Speak clearly to the patient in simple sentences, drug is producing its therapeutic effect, the nurse
use gestures or pictures when able should consider which finding most significant?
c) Speak slowly and clearly to assist the patient in a) Elevated blood pressure
forming the sounds b) Decreased level of consciousness (LOC)
d) Encourage the patient to repeat sounds of the c) Increased urine output
alphabet d) Decreased heart rate

Which of the following is the initial diagnostic test Which of the following is a contraindication for the
for a stroke? administration of tissue plasminogen activator
a) Noncontrast CT scan (t-PA)?
b) Transcranial Doppler studies a) Systolic blood pressure less than or equal to 185
c) ECG mm Hg
d) Carotid Doppler b) Ischemic stroke
c) Intracranial hemorrhage
A patient has been diagnosed as having global d) Age 18 years of age or older
aphasia. The nurse recognizes that the patient will
be unable to do which of the following actions? 1. A patient has had an ischemic stroke and has
a) Form words that are understandable been admitted to the medical unit. What action
b) Speak at all should the nurse perform to best prevent joint
c) Form words that are understandable or deformities?
comprehend the spoken word A) Place the patient in the prone position for 30
d) Comprehend the spoken word minutes/day.
B) Assist the patient in acutely flexing the thigh to
A patient is admitted via ambulance to the promote movement.
emergency room of a stroke center at 1:30 p.m. C) Place a pillow in the axilla when there is limited
with symptoms that the patient said began at 1:00 external rotation.
p.m. Within 1 hour, an ischemic stroke had been D) Place patient's hand in pronation.
A patient who just suffered a suspected ischemic
A patient diagnosed with transient ischemic attacks stroke is brought to the ED by ambulance. On what
(TIAs) is scheduled for a carotid endarterectomy. should the nurse's primary assessment focus?
The nurse explains that this procedure will be done A) Cardiac and respiratory status
for what purpose? B) Seizure activity
A) To decrease cerebral edema C) Pain
B) To prevent seizure activity that is common D) Fluid and electrolyte balance
following a TIA
C) To remove atherosclerotic plaques blocking A patient with a cerebral aneurysm exhibits signs
cerebral flow and symptoms of an increase in intracranial
D) To determine the cause of the TIA pressure (ICP). What nursing intervention would be
most appropriate for this patient?
The nurse is discharging home a patient who A) Range-of-motion exercises to prevent
suffered a stroke. He has a flaccid right arm and leg contractures
and is experiencing problems with urinary B) Encouraging independence with ADLs to
incontinence. The nurse makes a referral to a home promote recovery
health nurse because of an awareness of what C) Early initiation of physical therapy
common patient response to a change in body D) Absolute bed rest in a quiet, nonstimulating
image? environment
A) Denial
B) Fear A patient recovering from a stroke has severe
C) Depression shoulder pain from subluxation of the shoulder and
D) Disassociation is being cared for on the unit. To prevent further
injury and pain, the nurse caring for this patient is
When caring for a patient who had a hemorrhagic aware of what principle of care?
stroke, close monitoring of vital signs and A) The patient should be fitted with a cast because
neurologic changes is imperative. What is the use of a sling should be avoided due to adduction
earliest sign of deterioration in a patient with a of the affected shoulder.
hemorrhagic stroke of which the nurse should be B) Elevation of the arm and hand can lead to
aware? further complications associated with edema.
A) Generalized pain C) Passively exercising the affected extremity is
B) Alteration in level of consciousness (LOC) avoided in order to minimize pain.
C) Tonic-clonic seizures D) The patient should be taught to interlace fingers,
D) Shortness of breath place palms together, and slowly bring scapulae
forward to avoid excessive force to shoulder.
The nurse is performing stroke risk screenings at a
hospital open house. The nurse has identified four The patient has been diagnosed with aphasia after
patients who might be at risk for a stroke. Which suffering a stroke. What can the nurse do to best
patient is likely at the highest risk for a hemorrhagic make the patient's atmosphere more conducive to
stroke? communication?
A) White female, age 60, with history of excessive A) Provide a board of commonly used needs and
alcohol intake phrases.
B) White male, age 60, with history of uncontrolled B) Have the patient speak to loved ones on the
hypertension phone daily.
C) Black male, age 60, with history of diabetes C) Help the patient complete his or her sentences.
D) Black male, age 50, with history of smoking D) Speak in a loud and deliberate voice to the
patient.
The nurse is assessing a patient with a suspected B) Take ibuprofen for complaints of a serious
stroke. What assessment finding is most headache.
suggestive of a stroke? C) Take antihypertensive medication as ordered.
A) Facial droop D) Drowsiness is normal for the first week after
B) Dysrhythmias discharge.
C) Periorbital edema
D) Projectile vomiting A patient diagnosed with a cerebral aneurysm
reports a severe headache to the nurse. What
The nurse is caring for a patient diagnosed with an action is a priority for the nurse?
ischemic stroke and knows that effective A) Sit with the patient for a few minutes.
positioning of the patient is important. Which of the B) Administer an analgesic.
following should be integrated into the patient's C) Inform the nurse-manager.
plan of care? D) Call the physician immediately.
A) The patient's hip joint should be maintained in a
flexed position. A patient is brought by ambulance to the ED after
B) The patient should be in a supine position unless suffering what the family thinks is a stroke. The
ambulating. nurse caring for this patient is aware that an
C) The patient should be placed in a prone position absolute contraindication for thrombolytic therapy is
for 15 to 30 minutes several times a day. what?
D) The patient should be placed in a Trendelenberg A) Evidence of hemorrhagic stroke
position two to three times daily to promote cerebral B) Blood pressure of 180/110 mm Hg
perfusion. C) Evidence of stroke evolution
D) Previous thrombolytic therapy within the past 12
A patient has been admitted to the ICU after being months
recently diagnosed with an aneurysm and the
patient's admission orders include specific When caring for a patient who has had a stroke, a
aneurysm precautions. What nursing action will the priority is reduction of ICP. What patient position is
nurse incorporate into the patient's plan of care? most consistent with this goal?
A) Elevate the head of the bed to 45 degrees. A) Head turned slightly to the right side
B) Maintain the patient on complete bed rest. B) Elevation of the head of the bed
C) Administer enemas when the patient is C) Position changes every 15 minutes while awake
constipated. D) Extension of the neck
D) Avoid use of thigh-high elastic compression
stockings. A patient who suffered an ischemic stroke now has
disturbed sensory perception. What principle
A nurse is caring for a patient diagnosed with a should guide the nurse's care of this patient?
hemorrhagic stroke. When creating this patient's A) The patient should be approached on the side
plan of care, what goal should be prioritized? where visual perception is intact.
A) Prevent complications of immobility. B) Attention to the affected side should be
B) Maintain and improve cerebral tissue perfusion. minimized in order to decrease anxiety.
C) Relieve anxiety and pain. C) The patient should avoid turning in the direction
D) Relieve sensory deprivation. of the defective visual field to minimize shoulder
subluxation.
The nurse is preparing health education for a D) The patient should be approached on the
patient who is being discharged after hospitalization opposite side of where the visual perception is
for a hemorrhagic stroke. What content should the intact to promote recovery.
nurse include in this education?
A) Mild, intermittent seizures can be expected.
What should be included in the patient's care plan A patient has recently begun mobilizing during the
when establishing an exercise program for a patient recovery from an ischemic stroke. To protect the
affected by a stroke? patient's safety during mobilization, the nurse
A) Schedule passive range of motion every other should perform what action?
day. A) Support the patient's full body weight with a
B) Keep activity limited, as the patient may be over waist belt during ambulation.
stimulated. B) Have a colleague follow the patient closely with
C) Have the patient perform active range-of-motion a wheelchair.
(ROM) exercises once a day. C) Avoid mobilizing the patient in the early morning
D) Exercise the affected extremities passively four or late evening.
or five times a day. D) Ensure that the patient's family members do not
participate in mobilization.
A female patient is diagnosed with a right-sided
stroke. The patient is now experiencing A patient diagnosed with a hemorrhagic stroke has
hemianopsia. How might the nurse help the patient been admitted to the neurologic ICU. The nurse
manage her potential sensory and perceptional knows that teaching for the patient and family
difficulties? needs to begin as soon as the patient is settled on
A) Keep the lighting in the patient's room low. the unit and will continue until the patient is
B) Place the patient's clock on the affected side. discharged. What will family education need to
C) Approach the patient on the side where vision is include?
impaired. A) How to differentiate between hemorrhagic and
D) Place the patient's extremities where she can ischemic stroke
see them. B) Risk factors for ischemic stroke
C) How to correctly modify the home environment
The public health nurse is planning a health D) Techniques for adjusting the patient's medication
promotion campaign that reflects current dosages at home
epidemiologic trends. The nurse should know that
hemorrhagic stroke currently accounts for what After a subarachnoid hemorrhage, the patient's
percentage of total strokes in the United States? laboratory results indicate a serum sodium level of
A) 43% less than 126 mEq/L. What is the nurse's most
B) 33% appropriate action?
C) 23% A) Administer a bolus of normal saline as ordered.
D) 13% B) Prepare the patient for thrombolytic therapy as
A patient who has experienced an ischemic stroke ordered.
has been admitted to the medical unit. The patient's C) Facilitate testing for hypothalamic dysfunction.
family in adamant that she remain on bed rest to D) Prepare to administer 3% NaCl by IV as
hasten her recovery and to conserve energy. What ordered.
principle of care should inform the nurse's response
to the family? A community health nurse is giving an educational
A) The patient should mobilize as soon as she is presentation about stroke and heart disease at the
physically able. local senior citizens center. What nonmodifiable risk
B) To prevent contractures and muscle atrophy, bed factor for stroke should the nurse cite?
rest should not exceed 4 weeks. A) Female gender
C) The patient should remain on bed rest until she B) Asian American race
expresses a desire to mobilize. C) Advanced age
D) Lack of mobility will greatly increase the patient's D) Smoking
risk of stroke recurrence.
A family member brings the patient to the clinic for
a follow-up visit after a stroke. The family member
asks the nurse what he can do to decrease his which cardiac dysrhythmia is associated with
chance of having another stroke. What would be cardiogenic embolic strokes?
the nurse's best answer? A) Ventricular tachycardia
A) "Have your heart checked regularly." B) Atrial fibrillation
B) "Stop smoking as soon as possible." C) Supraventricular tachycardia
C) "Get medication to bring down your sodium D) Bundle branch block
levels."
D) "Eat a nutritious diet." The pathophysiology of an ischemic stroke involves
the ischemic cascade, which includes the following
The nurse is reviewing the medication steps:
administration record of a female patient who 1. Change in pH
possesses numerous risk factors for stroke. Which 2. Blood flow decreases
of the woman's medications carries the greatest 3. A switch to anaerobic respiration
potential for reducing her risk of stroke? 4. Membrane pumps fail
A) Naproxen 250 PO b.i.d. 5. Cells cease to function
B) Calcium carbonate 1,000 mg PO b.i.d. 6. Lactic acid is generated
C) Aspirin 81 mg PO o.d.
D) Lorazepam 1 mg SL b.i.d. PRN Put these steps in order in which they occur.
A) 635241
A nurse in the ICU is providing care for a patient B) 352416
who has been admitted with a hemorrhagic stroke. C) 236145
The nurse is performing frequent neurologic D) 162534
assessments and observes that the patient is
becoming progressively more drowsy over the As a member of the stroke team, the nurse knows
course of the day. What is the nurse's best that thrombolytic therapy carries the potential for
response to this assessment finding? benefit and for harm. The nurse should be
A) Report this finding to the physician as an cognizant of what contraindications for thrombolytic
indication of decreased metabolism. therapy? Select all that apply.
B) Provide more stimulation to the patient and A) INR above 1.0
monitor the patient closely. B) Recent intracranial pathology
C) Recognize this as the expected clinical course of C) Sudden symptom onset
a hemorrhagic stroke. D) Current anticoagulation therapy
D) Report this to the physician as a possible sign of E) Symptom onset greater than 3 hours prior to
clinical deterioration. admission

Following diagnostic testing, a patient has been After a major ischemic stroke, a possible
admitted to the ICU and placed on cerebral complication is cerebral edema. Nursing care
aneurysm precautions. What nursing action should during the immediate recovery period from an
be included in patient's plan of care? ischemic stroke should include which of the
A) Supervise the patient's activities of daily living following?
closely. A) Positioning to avoid hypoxia
B) Initiate early ambulation to prevent complications B) Maximizing PaCO2
of immobility. C) Administering hypertonic IV solution
C) Provide a high-calorie, low-protein diet. D) Initiating early mobilization
D) Perform all of the patient's hygiene and feeding.
The nurse is caring for a patient recovering from an
A preceptor is discussing stroke with a new nurse ischemic stroke. What intervention best addresses
on the unit. The preceptor would tell the new nurse a potential complication after an ischemic stroke?
A) Providing frequent small meals rather than three B) "The focus on care in a rehabilitation facility is to
larger meals help the patient to resume as much self-care as
B) Teaching the patient to perform deep breathing possible."
and coughing exercises C) "We aren't here to care for her the way the
C) Keeping a urinary catheter in situ for the full hospital staff did; we are here to help her get better
duration of recovery so she can go home."
D) Limiting intake of insoluble fiber D) "Rehabilitation means helping patients do
exactly what they did before their stroke."
During a patient's recovery from stroke, the nurse
should be aware of predictors of stroke outcome in A patient with a new diagnosis of ischemic stroke is
order to help patients and families set realistic deemed to be a candidate for treatment with tissue
goals. What are the predictors of stroke outcome? plasminogen activator (t-PA) and has been
Select all that apply. admitted to the ICU. In addition to closely
A) National Institutes of Health Stroke Scale monitoring the patient's cardiac and neurologic
(NIHSS) score status, the nurse monitors the patient for signs of
B) Race what complication?
C) LOC at time of admission A) Acute pain
D) Gender B) Septicemia
E) Age C) Bleeding
D) Seizures
A nursing student is writing a care plan for a newly
admitted patient who has been diagnosed with a 1. A nurse is communicating with a client who has
stroke. What major nursing diagnosis should most aphasia after having a stroke. Which action should
likely be included in the patient's plan of care? the nurse take?
A) Adult failure to thrive a. Use one long sentence to say everything that
B) Post-trauma syndrome needs to be said.
C) Hyperthermia b. Keep the television on while she speaks.
D) Disturbed sensory perception c. Talk in a louder than normal voice.
d. Face the client and establish eye contact.
When preparing to discharge a patient home, the
nurse has met with the family and warned them that 2. A client who has experienced an initial transient
the patient may exhibit unexpected emotional ischemic attack (TIA) states: "I'm glad it wasn't
responses. The nurse should teach the family that anything serious." Which is the best nursing
these responses are typically a result of what response to this statement?
cause? a. "I sense that you are happy it was not a stroke".
A) Frustration around changes in function and b. "People who experience a TIA will develop a
communication stroke".
B) Unmet physiologic needs c. "TIA symptoms are short-lived and resolve within
C) Changes in brain activity during sleep and 24 hours".
wakefulness d. "TIA is a warning sign. Let's talk about lowering
D) Temporary changes in metabolism your risks."

A rehabilitation nurse caring for a patient who has 3. When caring for a patient who had a
had a stroke is approached by the patient's family hemorrhagic stroke, close monitoring of vital signs
and asked why the patient has to do so much for and neurologic changes is imperative. What is the
herself when she is obviously struggling. What earliest sign of deterioration in a patient with a
would be the nurse's best answer? hemorrhagic stroke of which the nurse should be
A) "We are trying to help her be as useful as she aware?
possibly can." a. Generalized pain
b. Alteration in level of consciousness (LOC) c. Recognize this as the expected clinical course of
c. Tonic-clonic seizures a hemorrhagic stroke.
d. Shortness of breath d. Report this to the physician as a possible sign of
clinical deterioration.
4. When caring for a patient who has had a stroke,
a priority is reduction of ICP. What patient position 9. An emergency department nurse is interviewing
is most consistent with this goal? a client with signs of an ischemic stroke that began
a. Head turned slightly to the right side 2 hours ago. The client reports that she had a
b. Elevation of the head of the bed cholecystectomy 6 weeks ago and is taking digoxin,
c. Position changes every 15 minutes while awake coumadin, and labetelol. This client is not eligible
d. Extension of the neck for thrombolytic therapy for which of the following
reasons?
5. A patient diagnosed with an ischemic stroke a. She is taking coumadin.
should be treated within the first 3 hours of b. She is not within the treatment time window.
symptom onset with which of the following? c. She is taking digoxin.
a. Clopidogrel d. She had surgery 6 weeks ago
b. Extended release dipyridamole
c. Tissue plasminogen activator (tPA) 10. From which direction should a nurse approach
d. Atorvastatin a client who is blind in the right eye?
6. A nurse is assisting with a community screening a. From directly in front of the client
for people at high risk for stroke. To which of the b. From the right side of the client
following clients would the nurse pay most c. From the left side of the client
attention? d. From directly behind the client
a. A 60-year-old African-American man
b. A 40-year-old Caucasian woman 11. Which of the following is a contraindication for
c. A 62-year-old Caucasian woman the administration of tissue plasminogen activator
d. A 28-year-old pregnant African-American woman (t-PA)?
a. Intracranial hemorrhage
7. The nurse is reviewing the medication b. Ischemic stroke
administration record of a female patient who c. Age 18 years of age or older
possesses numerous risk factors for stroke. Which d. Systolic blood pressure less than or equal to 185
of the woman's medications carries the greatest mm Hg
potential for reducing her risk of stroke?
a. Naproxen 250 PO b.i.d. 12. A client with a history of atrial fibrillation has
b. Calcium carbonate 1,000 mg PO b.i.d. experienced a TIA. In an effort to reduce the risk of
c. Aspirin 81 mg PO o.d. cerebrovascular accident (CVA), the nurse
d. Lorazepam 1 mg SL b.i.d. PRN anticipates the medical treatment to include which
of the following?
8. A nurse in the ICU is providing care for a patient a. Cholesterol-lowering drugs
who has been admitted with a hemorrhagic stroke. b. Anticoagulant therapy
The nurse is performing frequent neurologic c. Monthly prothrombin levels
assessments and observes that the patient is d. Carotid endarterectomy
becoming progressively more drowsy over the
course of the day. What is the nurse's best 13. A patient is exhibiting classic signs of a
response to this assessment finding? hemorrhagic stroke. What complaint from the
a. Report this finding to the physician as an patient would be an indicator of this type of
indication of decreased metabolism. stroke13%
b. Provide more stimulation to the patient and a. Numbness of an arm or leg
monitor the patient closely. b. Double vision
c. Severe headache lower extremity. What ordered medication does the
d. Dizziness and tinnitus nurse administer to help alleviate this problem?
14. A 154-pound woman has been prescribed tPA a. Diphenhydramine (Benadryl)
(0.9 mg/kg) for an ischemic stroke. The nurse b. Lioresal (Baclofen)
knows to give how many mg initially? c. Heparin
a. 6.3 mg d. Pregabalin (Lyrica)
b. 7.5 mg
c. 8.3 mg In promoting health maintenance for prevention of
d. 10 mg strokes, the nurse understands that the highest risk
for the most common type of stroke is present in
15. What clinical manifestations does the nurse which people?
recognize when a patient has had a right a. blacks
hemispheric stroke? b. women who smoke
a. Left visual field deficit c. persons with hypertension and diabetes
b. Aphasia d. those who are obese with high dietary fat intake
c. Slow, cautious behavior
d. Altered intellectual ability a thrombus that develops in a cerebral artery does
not always cause a loss of neurologic function
16. A physician orders several drugs for a client because
with hemorrhagic stroke. Which drug order should a. the body can dissolve atherosclerotic plaques as
the nurse question? they form
a. Heparin sodium b. some tissues of the brain do not require constant
b. Dexamethasone (Decadron) blood supply to prevent damage
c. Methyldopa (Aldomet) c. circulation via the circle of willis may provide
d. Phenytoin (Dilantin) blood supply to the affected area of the brain
d. neurologic deficits occur only when major
17. The nurse is caring for a patient diagnosed with arteries are occluded by thrombus formation
a hemorrhagic stroke. The nurse recognizes that around atherosclerotic plaque
which of the following interventions is most
important? a patient comes to the ED with numbest of the face
a. Elevating the head of the bed at 30 degrees and an inability to speak. while the patient awaits
b. Monitoring for seizure activity examination, the symptoms disappear and the
c. Administering a stool softener patient requests discharge. why should the nurse
d. Maintaining a patent airway emphasize that it is important for the patient to be
treated before leaving
18. The nurse is performing stroke risk screenings a. the patient has probably experience an
at a hospital open house. The nurse has identified asymptotic lacunar stroke
four patients who might be at risk for a stroke. b. the symptoms are likely to return and progress to
Which patient is likely at the highest risk for a worsening neurologic deficit in the next 24 hours
hemorrhagic stroke? c. neurologic deficits that are transient occur most
a. White female, age 60, with history of excessive often as a result of small hemorrhages that clot off
alcohol intake d. the patient has probably had a transient ischemic
b. White male, age 60, with history of uncontrolled attack (TIA) which is a sign of progressive
hypertension cerebrovascular disease
c. Black male, age 60, with history of diabetes
d. Black male, age 50, with history of smoking which statement describe characteristics of a stroke
caused by an intracerebral hemorrhage> (select all
19. A patient who has suffered a stroke begins that apply)?
having complications regarding spasticity in the a. carries poor prognosis
b. caused by rupture of a vessel
c. strong association with hypertension The incidence of ischemic stroke in patients with
d. commonly occurs during or after sleep TIAs and other risk factors is reduced with
e. creates a mass that compresses the brain administration of
a. nimodipine (Nimotop)
which type of stroke is associated with endocardial b. furosemide (Lasix)
disorders, has a rapid onset and is likely to occur c. warfarin (Coumadin)
during activity? d. daily low dose aspirin
a. embolic
b. thrombotic
c. intracerebral hemorrhage
d. subarachnoid hemorrhage what is the priority intervention in the ED for the
patient with a stroke?
what primarily determines the neurological a. IV fluid replacement
functions that are affected by a stroke? b. giving osmotic diuretics to reduce cerebral
a. the amount of tissue area involved edema
b. the rapidity of the onset of symptoms c. starting hypothermia to decrease the oxygen
c. the brain area perfused by the affected artery needs of the brain
d. the presence or absence of collateral circulation d.maintaining respiratory function with a patent
airway and oxygen administration
the patient has a lack of comprehension of both
verbal and written language. which type of A diagnosis of a ruptured cerebral aneurysm has
communication difficulty does this patient. have? been made in a patient with manifestations of a
a. dysarthria stroke. The nurse anticipates that treatment options
b. fluent dysphasia that would be evaluated for the patient include
c. receptive aphasia a. hyperventilation therapy
d. expressive aphasia b. surgical clipping of the aneurysm
c. administration of hyperosmotic agents
A patient is admitted to the hospital with a left d. administration of thrombolytic therapy
hemiplegia. To determine the size and location and
to ascertain whether a stroke is ischemic or During the acute phase of a stroke, the nurse
hemorrhagic, the nurse anticipates that the health assesses the patient's vital signs and neurologic
care provider will request a status every 4 hours. A cardiovascular sign that the
a. lumbar puncture nurse would see as the body attempts to increase
b. cerebral angiography cerebral blood flow is
c. MRI a. hypertension
d. CT scan with contrast b. fluid overload
c. cardiac dysrhythmias
A carotid endarterectomy is being considered as a d. S3 and S4 heart sounds
treatment for a patient who has had several TIAs.
The nurse explains to the patient that this surgery During the secondary assessment of a patient with
a. involves intracranial surgery to join a superficial a stroke, what should be included (select all that
extracranial artery to an intracranial artery apply)?
b. is used to restore blood to the brain following an a. gaze
obstruction of a cerebral artery b. sensation
c. is used to open a stenosis in a carotid artery with c. facial palsy
a balloon and stent to restore cerebral circulation d. proprioception
d. involves removing an atherosclerotic plaque in e. current medications
the carotid artery to prevent an impending stroke f. distal motor function
b. " not everyone is eligible for this drug. has he
what is a nursing intervention that is indicated for had surgery lately?"
the patient with hemiplegia? c. "you should discuss the treatment of your
a. the use of a footboard to prevent plantar flexion husband with his doctors"
b. immobilization of the affected arm against the d. "the medication you are talking about dissolves
chest with a sling clots and could cause more bleeding in your
c. positioning the patient in bed with each joint husbands brain"
lower than the joint proximal to it
d. having the patient perform passive rang of the rehab nurse assesses the patient, caregiver
motion (ROM) of the affected limb with the and family before planning the rehab program for
unaffected limb this patient. what must be included in this
assessment (select all that apply)?
a newly admitted patient diagnosed with a right a. cognitive status of the family
sided brain stroke has homonymous hemianopsia. b. patient resources and support
early in the case of the patient what should the c. physical status of all body systems
nurse do ? d. rehab potential of patient
a. place objects on the right side within the patients e. body strength remaining after the stroke
field of vision f. patient and caregiver expectation of the rehab
b. approach the patient from the left side to assess
the patients ability to compensate what is an appropriate nursing intervention to
c. place objects on the left side to assess the promote communication during rehab of the patient
patients ability to compensate with aphasia?
d. patch the affected eye to encourage patient to a. allow time for the individual to complete their
turn the head to scan the environments thoughts
b. use gestures, pic utters, and music to stimulate
four days following a stroke, a patient is to start oral patients responses
fluids and feedings. before feeding the patient, what c. structure statements so that the patient does not
should the nurse do first? have to respond verbally
a. check the patient's gag reflex d. use flashcards with simple words and pictures to
b. order a soft diet for the paitnet promote recall of language
c. raise the head of the bed to a sitting position
d. assess the patients ability to swallow tiny A patient with right hemisphere stroke has
amounts of crushed ice unilateral neglect . During the patient's
rehabilitation, it is important for the nurse to
what is an appropriate food for a patient with a a. avoid positioning the patient on the affected side
stroke who has mild dysphagia? b. place all objects for care on the patient's
a. fruit juices unaffected side
b. pureed meats c. teach the patient to care consciously for the
c. scrambled eggs affected side
d. fortified milkshakes d. protect the affected side from injury with pillows
and supports
a patients wife asks the nurse why her husband did
not receive the clot-busting medication (tissue A patient with a stroke has a right sided hemiplegia.
plasminogen activators [tPA]) she has been riding The nurse prepares family members to help control
about. her husband is diagnosed with a behavior changes seen with this type of stroke by
hemorrhagic stroke. what is the best response by teaching them to
the nurse to the patients wife? a. ignore undesirable behaviors manifested by the
a. " he didn't arrive within the time frame for that patient
therapy"
b. provide directions to the patient verbally in small A. Shifting cerebrospinal fluid to other areas of the
steps brain and spinal cord
c. distract the patient from inappropriate emotional B. Vasodilation of cerebral vessels
responses C. Decreasing cerebrospinal fluid production
d. supervise all activities before allowing the patient D. Leaking proteins into the brain barrier
to pursue them independently
3. A patient is being treated for increased
the nurse can best assist the patient and family in intracranial pressure. Which activities below should
coping with the long-term effects of a stroke by the patient avoid performing?
doing what? A. Coughing
a. informing family members that the patient will B. Sneezing
need assistance with almost all ADLs C. Talking
b. explain that the patents prestroke behavior will D. Valsalva maneuver
return as improvement progresses E. Vomiting
c. encouraging the patient and family members to F. Keeping the head of the bed between 30- 35
seek assistance from family therapy or stroke degrees
support groups
d. helping the patient and family understand the 4. A patient is experiencing hyperventilation and
significance of residual stroke damage to promote has a PaCO2 level of 52. The patient has an ICP of
problem solving and planning 20 mmHg. As the nurse you know that the PaCO2
level will?
which intervention can the nurse delegate to the A. cause vasoconstriction and decrease the ICP
licenses practical nurse when caring for a patient B. promote diuresis and decrease the ICP
following an acute stroke C. cause vasodilation and increase the ICP
a. assess the patients neurological status D. cause vasodilation and decrease the ICP
b. assess the patients gag reface before beginning
feeding 5. You're providing education to a group of nursing
c. administer ordered antihypertensives and platelet students about ICP. You explain that when cerebral
inhibitors perfusion pressure falls too low the brain is not
d. teach the patients caregivers strategies to properly perfused and brain tissue dies. A student
minimize unilateral neglect asks, "What is a normal cerebral perfusion pressure
level?" Your response is:
1.6 Increased ICP A. 5-15 mmHg
1. Select the main structures below that play a role B. 60-100 mmHg
with altering intracranial pressure: C. 30-45 mmHg
A. Brain D. >160 mmHg
B. Neurons
C. Cerebrospinal Fluid 6. Which patient below is at MOST risk for
D. Blood increased intracranial pressure?
E. Periosteum A. A patient who is experiencing severe
F. Dura mater hypotension.
B. A patient who is admitted with a traumatic brain
2. The Monro-Kellie hypothesis explains the injury.
compensatory relationship among the structures in C. A patient who recently experienced a myocardial
the skull that play a role with intracranial pressure. infarction.
Which of the following are NOT compensatory D. A patient post-op from eye surgery.
mechanisms performed by the body to decrease
intracranial pressure naturally? Select all that apply: 7. A patient with increased ICP has the following
vital signs: blood pressure 99/60, HR 65,
Temperature 101.6 'F, respirations 14, oxygen B. Tachycardia
saturation of 95%. ICP reading is 21 mmHg. Based C. Decrease in pulse pressure
on these findings you would? D. Cheyne-stokes
A. Administered PRN dose of a vasopressor E. Hemiplegia
B. Administer 2 L of oxygen F. Decerebrate posturing
C. Remove extra blankets and give the patient a
cool bath 14. You're maintaining an external ventricular drain.
D. Perform suctioning The ICP readings should be?
A. 5 to 15 mmHg
8. A patient has a ventriculostomy. Which finding B. 20 to 35 mmHg
would you immediately report to the doctor? C. 60 to 100 mmHg
A. Temperature 98.4 'F D. 5 to 25 mmHg
B. CPP 70 mmHg
C. ICP 24 mmHg 15. Which patient below with ICP is experiencing
D. PaCO2 35 Cushing's Triad? A patient with the following:
A. BP 150/112, HR 110, RR 8
9. External ventricular drains monitor ICP and are B. BP 90/60, HR 80, RR 22
inserted where? C. BP 200/60, HR 50, RR 8
A. Subarachnoid space D. BP 80/40, HR 49, RR 12
B. Lateral Ventricle
C. Epidural space 16. The patient has a blood pressure of 130/88 and
D. Right Ventricle ICP reading of 12. What is the patient's cerebral
perfusion pressure, and how do you interpret this
10. Which of the following is contraindicated in a as the nurse?
patient with increased ICP? A. 90 mmHg, normal
A. Lumbar puncture B. 62 mmHg, abnormal
B. Midline position of the head C. 36 mmHg, abnormal
C. Hyperosmotic diuretics D. 56 mmHg, normal
D. Barbiturates medications
17. According to question 16, the patient's blood
11. You're collecting vital signs on a patient with pressure is 130/88. What is the patient's mean
ICP. The patient has a Glascoma Scale rating of 4. arterial pressure (MAP)?
How will you assess the patient's temperature? A. 42
A. Rectal B. 74
B. Oral C. 102
C. Axillary D. 88

12. A patient who experienced a cerebral 18. During the assessment of a patient with
hemorrhage is at risk for developing increased ICP. increased ICP, you note that the patient's arms are
Which sign and symptom below is the EARLIEST extended straight out and toes pointed downward.
indicator the patient is having this complication? You will document this as:
A. Bradycardia A. Decorticate posturing
B. Decerebrate posturing B. Decerebrate posturing
C. Restlessness C. Flaccid posturing
D. Unequal pupil size

13. Select all the signs and symptoms that occur 19. While positioning a patient in bed with
with increased ICP: increased ICP, it important to avoid?
A. Decorticate posturing A. Midline positioning of the head
B. Placing the HOB at 30-35 degrees insulin IV bolus and then to start an insulin drip per
C. Preventing flexion of the neck protocol. The patient's labs are the following: pH
D. Flexion of the hips 7.25, Glucose 455, potassium 2.5. Which of the
following is the most appropriate nursing
20. During the eye assessment of a patient with intervention to perform next?*
increased ICP, you need to assess the A. Start the IV fluids and administer the insulin
oculocephalic reflex. If the patient has brain stem bolus and drip as ordered
damage what response will you find? B. Hold the insulin and notify the doctor of the
A. The eyes will roll down as the head is moved potassium level of 2.5
side to side. C. Hold IV fluids and administer insulin as ordered
B. The eyes will move in the opposite direction as D. Recheck the glucose level
the head is moved side to side.
C. The eyes will roll back as the head is moved Which patient is MOST likely to develop Diabetic
side to side. Ketoacidosis?*
D. The eyes will be in a fixed mid-line position as A. A 25 year old female newly diagnosed with
the head is moved side to side. Cushing's Disease taking glucocorticoids.
B. A 36 year old male with diabetes mellitus who
21. A patient is receiving Mannitol for increased has been unable to eat the past 2 days due to a
ICP. Which statement is INCORRECT about this gastrointestinal illness and has been unable to take
medication? insulin.
A. Mannitol will remove water from the brain and C. A 35 year old female newly diagnosed with Type
place it in the blood to be removed from the body. 2 diabetes.
B. Mannitol will cause water and electrolyte D. None of the options are correct.
reabsorption in the renal tubules.
C. When a patient receives Mannitol the nurse Which of the following statements are INCORRECT
must monitor the patient for both fluid volume about Diabetic Ketoacidoisis?*
overload and depletion. A. Extreme Hyperglycemia that presents with blood
D. Mannitol is not for patients who are experiencing glucose >600 mg/dL
anuria. B. Ketones are present in the urine
C. Metabolic acidosis is present with Kussmaul
22. What assessment finding requires immediate breathing
intervention if found while a patient is receiving D. Potassium levels should be at least 3.3 or higher
Mannitol? during treatment of DKA with insulin therapy
A. An ICP of 10 mmHg
B. Crackles throughout lung fields You are providing care to a patient experiencing
C. BP 110/72 diabetic ketoacidosis. The patient is on an insulin
D. Patient complains of dry mouth and thirst drip and their current glucose level is 300. In
addition to this, the patient also has 5% Dextrose
1.7 Diabetic Ketoacidosis 0.45% NS infusing in the right antecubital vein.
Which of the following is not a sign or symptom of Which of the following patient signs/symptoms
Diabetic Ketoacidosis?* causes concern?*
A. Positive Ketones in the urine A. Patient complains of thirst.
B. Oliguria B. Patient has a potassium level of 2.3
C. Polydipsia C. Patient's skin and mucous membranes are dry.
D. Abdominal Pain D. Patient is nauseous.

A patient is admitted with Diabetic Ketoacidosis. What type of insulin do you expect the doctor to
The physician orders intravenous fluids of 0.9% order for treatment of DKA?*
Normal Saline and 10 units of intravenous regular A. IV Novolog
B. IV Levemir On admission to the burn unit, a patient with an
C. IV NPH approximate 25% total body surface area (TBSA)
D. IV Regular Insulin burn has the following initial laboratory results: Hct
58%, Hgb 18.2 mg/dL (172 g/L), serum K+ 4.9
A patient diagnosed with diabetes mellitus is being mEq/L (4.8 mmol/L), and serum Na+ 135 mEq/L
discharged home and you are teaching them about (135 mmol/L). Which action will the nurse anticipate
preventing DKA. What statement by the patient taking now?
demonstrates they understood your teaching about a. Monitor urine output every 4 hours.
this condition?* b. Continue to monitor the laboratory results.
A. "I should not be alarmed if ketones are present c. Increase the rate of the ordered IV solution.
in my urine because this is expected during illness." d. Type and crossmatch for a blood transfusion.
B. "It is normal for my blood sugar to be 250-350
mg/dL while I'm sick." A patient is admitted to the burn unit with burns to
C. "I will hold off taking my insulin while I'm sick." the head, face, and hands. Initially, wheezes are
D. "It is important I check my blood glucose every heard, but an hour later, the lung sounds are
3-4 hours when I'm sick and consume liquids." decreased and no wheezes are audible. What is
the best action for the nurse to take?
The nurse assisting in the admission of a client with a. Encourage the patient to cough and auscultate
diabetic ketoacidosis will anticipate the physician the lungs again.
ordering which of the following types of intravenous b. Notify the health care provider and prepare for
solution if the client cannot take any fluids orally? " endotracheal intubation.
a. 0.45% normal saline solution c. Document the results and continue to monitor the
b. Lactated Ringer's solution patient's respiratory rate.
c. 0.9 normal saline solution d. Reposition the patient in high-Fowler's position
d. 5% dextrose in water (D5W and reassess breath sounds.
A. 0.45% normal saline solution
A patient with severe burns has crystalloid fluid
Which of the following statements are INCORRECT replacement ordered using the Parkland formula.
about Diabetic Ketoacidosis? The initial volume of fluid to be administered in the
A. Extreme Hyperglycemia that presents with blood first 24 hours is 30,000 mL. The initial rate of
glucose >600 mg/dL administration is 1875 mL/hr. After the first 8 hours,
B. Ketones are present in the urine what rate should the nurse infuse the IV fluids?
C. Metabolic acidosis is present with Kussmaul a. 350 mL/hour
breathing b. 523 mL/hour
D. Potassium levels should be at least 3.3 or higher c. 938 mL/hour
during treatment of DKA with insulin therapy d. 1250 mL/hour

1.8 Extensive Burns During the emergent phase of burn care, which
When assessing a patient who spilled hot oil on the assessment will be most useful in determining
right leg and foot, the nurse notes that the skin is whether the patient is receiving adequate fluid
dry, pale, hard skin. The patient states that the burn infusion?
is not painful. What term would the nurse use to a. Check skin turgor.
document the burn depth? b. Monitor daily weight.
a. First-degree skin destruction c. Assess mucous membranes.
b. Full-thickness skin destruction d. Measure hourly urine output.
c. Deep partial-thickness skin destruction
d. Superficial partial-thickness skin destruction A patient has just been admitted with a 40% total
body surface area (TBSA) burn injury. To maintain
adequate nutrition, the nurse should plan to take b. Stool frequency
which action? c. Abdominal distention
a. Insert a feeding tube and initiate enteral d. Stools for occult blood
feedings.
b. Infuse total parenteral nutrition via a central The nurse is reviewing the medication
catheter. administration record (MAR) on a patient with
c. Encourage an oral intake of at least 5000 kcal partial-thickness burns. Which medication is best
per day. for the nurse to administer before scheduled wound
d. Administer multiple vitamins and minerals in the debridement?
IV solution. a. Ketorolac (Toradol)
b. Lorazepam (Ativan)
While the patient's full-thickness burn wounds to c. Gabapentin (Neurontin)
the face are exposed, what is the best nursing d. Hydromorphone (Dilaudid)
action to prevent cross contamination?
a. Use sterile gloves when removing old dressings. A young adult patient who is in the rehabilitation
b. Wear gowns, caps, masks, and gloves during all phase after having deep partial-thickness face and
care of the patient. neck burns has a nursing diagnosis of disturbed
c. Administer IV antibiotics to prevent bacterial body image. Which statement by the patient
colonization of wounds. indicates that the problem is resolving?
d. Turn the room temperature up to at least 70° F a. "I'm glad the scars are only temporary."
(20° C) during dressing changes. b. "I will avoid using a pillow, so my neck will be
OK."
A nurse is caring for a patient who has burns of the c. "I bet my boyfriend won't even want to look at me
ears, head, neck, and right arm and hand. The anymore."
nurse should place the patient in which position? d. "Do you think dark beige makeup foundation
a. Place the right arm and hand flexed in a position would cover this scar on my cheek?"
of comfort.
b. Elevate the right arm and hand on pillows and The nurse caring for a patient admitted with burns
extend the fingers. over 30% of the body surface assesses that urine
c. Assist the patient to a supine position with a output has dramatically increased. Which action by
small pillow under the head. the nurse would best ensure adequate kidney
d. Position the patient in a side-lying position with function?
rolled towel under the neck. a. Continue to monitor the urine output.
b. Monitor for increased white blood cells (WBCs).
A patient with circumferential burns of both legs c. Assess that blisters and edema have subsided.
develops a decrease in dorsalis pedis pulse d. Prepare the patient for discharge from the burn
strength and numbness in the toes. Which action unit.
should the nurse take?
a. Notify the health care provider. A patient with burns covering 40% total body
b. Monitor the pulses every 2 hours. surface area (TBSA) is in the acute phase of burn
c. Elevate both legs above heart level with pillows. treatment. Which snack would be best for the nurse
d. Encourage the patient to flex and extend the toes to offer to this patient?
on both feet. a. Bananas
b. Orange gelatin
Esomeprazole (Nexium) is prescribed for a patient c. Vanilla milkshake
who incurred extensive burn injuries 5 days ago. d. Whole grain bagel
Which nursing assessment would best evaluate the
effectiveness of the medication? A patient has just arrived in the emergency
a. Bowel sounds department after an electrical burn from exposure
to a high-voltage current. What is the priority a. Blood pressure is 95/48 per arterial line.
nursing assessment? b. Serous exudate is leaking from the burns.
a. Oral temperature c. Cardiac monitor shows a pulse rate of 108.
b. Peripheral pulses d. Urine output is 20 mL per hour for the past 2
c. Extremity movement hours.
d. Pupil reaction to light
Which patient should the nurse assess first?
An employee spills industrial acids on both arms a. A patient with smoke inhalation who has
and legs at work. What is the priority action that the wheezes and altered mental status
occupational health nurse at the facility should b. A patient with full-thickness leg burns who has a
take? dressing change scheduled
a. Remove nonadherent clothing and watch. c. A patient with abdominal burns who is
b. Apply an alkaline solution to the affected area. complaining of level 8 (0 to 10 scale) pain
c. Place cool compresses on the area of exposure. d. A patient with 40% total body surface area
d. Cover the affected area with dry, sterile (TBSA) burns who is receiving IV fluids at 500
dressings. mL/hour

A patient who has burns on the arms, legs, and Which patient is most appropriate for the burn unit
chest from a house fire has become agitated and charge nurse to assign to a registered nurse (RN)
restless 8 hours after being admitted to the who has floated from the hospital medical unit?
hospital. Which action should the nurse take first? a. A 34-year-old patient who has a weight loss of
a. Stay at the bedside and reassure the patient. 15% from admission and requires enteral feedings.
b. Administer the ordered morphine sulfate IV. b. A 67-year-old patient who has blebs under an
c. Assess orientation and level of consciousness. autograft on the thigh and has an order for bleb
d. Use pulse oximetry to check the oxygen aspiration
saturation. c. A 46-year-old patient who has just come back to
the unit after having a cultured epithelial autograft
A patient arrives in the emergency department with to the chest
facial and chest burns caused by a house fire. d. A 65-year-old patient who has twice-daily burn
Which action should the nurse take first? debridements and dressing changes to
a. Auscultate the patient's lung sounds. partial-thickness facial burns
b. Determine the extent and depth of the burns.
c. Infuse the ordered lactated Ringer's solution. A patient who was found unconscious in a burning
d. Administer the ordered hydromorphone house is brought to the emergency department by
(Dilaudid). ambulance. The nurse notes that the patient's skin
color is bright red. Which action should the nurse
A patient with extensive electrical burn injuries is take first?
admitted to the emergency department. Which a. Insert two large-bore IV lines.
prescribed intervention should the nurse implement b. Check the patient's orientation.
first? c. Assess for singed nasal hair and dark oral
a. Assess oral temperature. mucous membranes.
b. Check a potassium level. d. Place the patient on 100% oxygen using a
c. Place on cardiac monitor. non-rebreather mask.
d. Assess for pain at contact points.
The nurse is reviewing laboratory results on a
Eight hours after a thermal burn covering 50% of a patient who had a large burn 48 hours ago. Which
patient's total body surface area (TBSA) the nurse result requires priority action by the nurse?
assesses the patient. Which information would be a a. Hematocrit 53%
priority to communicate to the health care provider? b. Serum sodium 147 mEq/L
c. Serum potassium 6.1 mEq/L
d. Blood urea nitrogen 37 mg/dL A patient is brought to the emergency department
from the site of a chemical fire, where he suffered a
The charge nurse observes the following actions burn that involves the epidermis, dermis, and the
being taken by a new nurse on the burn unit. Which muscle and bone of the right arm. On inspection,
action by the new nurse would require an the skin appears charred. Based on these
intervention by the charge nurse? assessment findings, what is the depth of the burn
a. The new nurse uses clean latex gloves when on the patient's arm?
applying antibacterial cream to a burn wound. A) Superficial partial-thickness
b. The new nurse obtains burn cultures when the B) Deep partial-thickness
patient has a temperature of 95.2° F (35.1° C). C) Full partial-thickness
c. The new nurse administers PRN fentanyl D) Full-thickness
(Sublimaze) IV to a patient 5 minutes before a
dressing change. The current phase of a patient's treatment for a
d. The new nurse calls the health care provider for burn injury prioritizes wound care, nutritional
a possible insulin order when a nondiabetic support, and prevention of complications such as
patient's serum glucose is elevated. infection. Based on these care priorities, the patient
is in what phase of burn care?
Which nursing action is a priority for a patient who A) Emergent
has suffered a burn injury while working on an B) Immediate resuscitative
electrical power line? C) Acute
a. Obtain the blood pressure. D) Rehabilitation
b. Stabilize the cervical spine.
c. Assess for the contact points. A patient in the emergent/resuscitative phase of a
d. Check alertness and orientation. burn injury has had blood work and arterial blood
gases drawn. Upon analysis of the patient's
Which action will the nurse include in the plan of laboratory studies, the nurse will expect the results
care for a patient in the rehabilitation phase after a to indicate what?
burn injury to the right arm and chest? A) Hyperkalemia, hyponatremia, elevated
a. Keep the right arm in a position of comfort. hematocrit, and metabolic acidosis
b. Avoid the use of sustained-release narcotics. B) Hypokalemia, hypernatremia, decreased
c. Teach about the purpose of tetanus hematocrit, and metabolic acidosis
immunization. C) Hyperkalemia, hypernatremia, decreased
d. Apply water-based cream to burned areas hematocrit, and metabolic alkalosis
frequently. D) Hypokalemia, hyponatremia, elevated
hematocrit, and metabolic alkalosis
A young adult patient who is in the rehabilitation
phase 6 months after a severe face and neck burn A patient has experienced an electrical burn and
tells the nurse, "I'm sorry that I'm still alive. My life has developed thick eschar over the burn site.
will never be normal again." Which response by the Which of the following topical antibacterial agents
nurse is best? will the nurse expect the physician to order for the
a. "Most people recover after a burn and feel wound?
satisfied with their lives." A) Silver sulfadiazine 1% (Silvadene) water-soluble
b. "It's true that your life may be different. What cream
concerns you the most?" B) Mafenide acetate 10% (Sulfamylon)
c. "It is really too early to know how much your life hydrophilic-based cream
will be changed by the burn." C) Silver nitrate 0.5% aqueous solution
d. "Why do you feel that way? You will be able to D) Acticoat
adapt as your recovery progresses."
An occupational health nurse is called to the floor of 25% of the body. After ensuring cardiopulmonary
a factory where a worker has sustained a flash burn stability, what would be the nurse's immediate,
to the right arm. The nurse arrives and the flames priority concern when planning this patient's care?
have been extinguished. The next step is to ìcool A) Fluid status
the burn.î How should the nurse cool the burn? B) Risk of infection
A) Apply ice to the site of the burn for 5 to 10 C) Nutritional status
minutes. D) Psychosocial coping
B) Wrap the patient's affected extremity in ice until
help arrives. The nurse is preparing the patient for mechanical
C) Apply an oil-based substance or butter to the dÈbridement and informs the patient that this will
burned area until help arrives. involve which of the following procedures?
D) Wrap cool towels around the affected extremity A) A spontaneous separation of dead tissue from
intermittently. the viable tissue
B) Removal of eschar until the point of pain and
An emergency department nurse has just admitted bleeding occurs
a patient with a burn. What characteristic of the C) Shaving of burned skin layers until bleeding,
burn will primarily determine whether the patient viable tissue is revealed
experiences a systemic response to this injury? D) Early closure of the wound
A) The length of time since the burn
B) The location of burned skin surfaces A patient with a partial-thickness burn injury had
C) The source of the burn Biobrane applied 2 weeks ago. The nurse notices
D) The total body surface area (TBSA) affected by that the Biobrane is separating from the burn
the burn wound. What is the nurse's most appropriate
intervention?
A nurse on a burn unit is caring for a patient in the A) Reinforce the Biobrane dressing with another
acute phase of burn care. While performing an piece of Biobrane.
assessment during this phase of burn care, the B) Remove the Biobrane dressing and apply a new
nurse recognizes that airway obstruction related to dressing.
upper airway edema may occur up to how long C) Trim away the separated Biobrane.
after the burn injury? D) Notify the physician for further
A) 2 days emergency-related orders.
B) 3 days
C) 5 days An emergency department nurse learns from the
D) 1 week paramedics that they are transporting a patient who
has suffered injury from a scald from a hot kettle.
A patient has sustained a severe burn injury and is What variables will the nurse consider when
thought to have an impaired intestinal mucosal determining the depth of burn?
barrier. Since this patient is considered at an A) The causative agent
increased risk for infection, what intervention will B) The patient's preinjury health status
best assist in avoiding increased intestinal C) The patient's prognosis for recovery
permeability and prevent early endotoxin D) The circumstances of the accident
translocation?
A) Early enteral feeding A nurse is caring for a patient who has sustained a
B) Administration of prophylactic antibiotics deep partial-thickness burn injury. In prioritizing the
C) Bowel cleansing procedures nursing diagnoses for the plan of care, the nurse
D) Administration of stool softeners will give the highest priority to what nursing
diagnosis?
A patient has been admitted to a burn intensive A) Activity Intolerance
care unit with extensive full-thickness burns over B) Anxiety
C) Ineffective Coping A) Sodium deficit
D) Acute Pain B) Decreased prothrombin time (PT)
C) Potassium deficit
A triage nurse in the emergency department (ED) D) Decreased hematocrit
receives a phone call from a frantic father who saw
his 4-year-old child tip a pot of boiling water onto A nurse is developing a care plan for a patient with
her chest. The father has called an ambulance. a partial-thickness burn, and determines that an
What would the nurse in the ED receiving the call appropriate goal is to maintain position of joints in
instruct the father to do? alignment. What is the best rationale for this
A) Cover the burn with ice and secure with a towel. intervention?
B) Apply butter to the area that is burned. A) To prevent neuropathies
C) Immerse the child in a cool bath. B) To prevent wound breakdown
D) Avoid touching the burned area under any C) To prevent contractures
circumstances. D) To prevent heterotopic ossification

A nurse is teaching a patient with a A patient's burns have required a homograft.


partial-thickness wound how to wear his elastic During the nurse's most recent assessment, the
pressure garment. How would the nurse instruct the nurse observes that the graft is newly covered with
patient to wear this garment? purulent exudate. What is the nurse's most
A) 4 to 6 hours a day for 6 months appropriate response?
B) During waking hours for 2 to 3 months after the A) Perform mechanical dÈbridement to remove the
injury exudate and prevent further infection.
C) Continuously B) Inform the primary care provider promptly
D) At night while sleeping for a year after the injury because the graft may need to be removed.
C) Perform range of motion exercises to increase
A patient is brought to the ED by paramedics, who perfusion to the graft site and facilitate healing.
report that the patient has partial-thickness burns D) Document this finding as an expected phase of
on the chest and legs. The patient has also graft healing.
suffered smoke inhalation. What is the priority in A nurse who is taking care of a patient with burns is
the care of a patient who has been burned and asked by a family member why the patient is losing
suffered smoke inhalation? so much weight. The patient is currently in the
A) Pain intermediate phase of recovery. What would be the
B) Fluid balance nurse's most appropriate response to the family
C) Anxiety and fear member?
D) Airway management A) ìHe's on a calorie-restricted diet in order to divert
energy to wound healing.î
A patient arrives in the emergency department after B) ìHis body has consumed his fat deposits for fuel
being burned in a house fire. The patient's burns because his calorie intake is lower than normal.î
cover the face and the left forearm. What extent of C) ìHe actually hasn't lost weight. Instead, there's
burns does the patient most likely have? been a change in the distribution of his body fat.î
A) 13% D) ìHe lost many fluids while he was being treated
B) 25% in the emergency phase of burn care.î
C) 9%
D) 18% A nurse has reported for a shift at a busy burns and
plastics unit in a large university hospital. Which
A nurse is caring for a patient in the patient is most likely to have life-threatening
emergent/resuscitative phase of burn injury. During complications?
this phase, the nurse should monitor for evidence A) A 4-year-old scald victim burned over 24% of the
of what alteration in laboratory values? body
B) A 27-year-old male burned over 36% of his body A patient is admitted to the burn unit after being
in a car accident transported from a facility 1000 miles away. The
C) A 39-year-old female patient burned over 18% of patient has burns to the groin area and
her body circumferential burns to both upper thighs. When
D) A 60-year-old male burned over 16% of his body assessing the patient's legs distal to the wound
in a brush fire site, the nurse should be cognizant of the risk of
what complication?
A patient is brought to the emergency department A) Ischemia
with a burn injury. The nurse knows that the first B) Referred pain
systemic event after a major burn injury is what? C) Cellulitis
A) Hemodynamic instability D) Venous thromboembolism (VTE)
B) Gastrointestinal hypermotility
C) Respiratory arrest A patient experienced a 33% TBSA burn 72 hours
D) Hypokalemia ago. The nurse observes that the patient's hourly
urine output has been steadily increasing over the
A patient with severe burns is admitted to the past 24 hours. How should the nurse best respond
intensive care unit to stabilize and begin fluid to this finding?
resuscitation before transport to the burn center. A) Obtain an order to reduce the rate of the
The nurse should monitor the patient closely for patient's IV fluid infusion.
what signs of the onset of burn shock? B) Report the patient's early signs of acute kidney
A) Confusion injury (AKI).
B) High fever C) Recognize that the patient is experiencing an
C) Decreased blood pressure expected onset of diuresis.
D) Sudden agitation D) Administer sodium chloride as ordered to
compensate for this fluid loss.
An emergency department nurse has just received
a patient with burn injuries brought in by A public health nurse has reviewed local data about
ambulance. The paramedics have started a the incidence and prevalence of burn injuries in the
large-bore IV and covered the burn in cool towels. community. These data are likely to support what
The burn is estimated as covering 24% of the health promotion effort?
patient's body. How should the nurse best address A) Education about home safety
the pathophysiologic changes resulting from major B) Education about safe storage of chemicals
burns during the initial burn-shock period? C) Education about workplace health threats
A) Administer IV fluids D) Education about safe driving
B) Administer broad-spectrum antibiotics
C) Administer IV potassium chloride A nurse is performing a home visit to a patient who
D) Administer packed red blood cells is recovering following a long course of inpatient
treatment for burn injuries. When performing this
A patient's burns are estimated at 36% of total body home visit, the nurse should do which of the
surface area; fluid resuscitation has been ordered following?
in the emergency department. After establishing A) Assess the patient for signs of electrolyte
intravenous access, the nurse should anticipate the imbalances.
administration of what fluid? B) Administer fluids as ordered.
A) 0.45% NaCl with 20 mEq/L KCl C) Assess the risk for injury recurrence.
B) 0.45% NaCl with 40 mEq/L KCl D) Assess the patient's psychosocial state.
C) Normal saline
D) Lactated Ringer's A patient has experienced burns to his upper thighs
and knees. Following the application of new wound
dressings, the nurse should perform what nursing B) ìThat is something for you to talk to your doctor
action? about because it's not a nursing responsibility.î
A) Instruct the patient to keep the wound site in a C) ìI know this is really important to you, but you
dependent position. have to realize that no one can make you look like
B) Administer PRN analgesia as ordered. you used to.î
C) Assess the patient's peripheral pulses distal to D) ìUnfortunately, it's likely that you will have most
the dressing. of these scars for the rest of your life.î
D) Assist with passive range of motion exercises to
ìsetî the new dressing. A patient who is in the acute phase of recovery
from a burn injury has yet to experience adequate
A nurse is caring for a patient with burns who is in pain control. What pain management strategy is
the later stages of the acute phase of recovery. The most likely to meet this patient's needs?
plan of nursing care should include which of the A) A patient-controlled analgesia (PCA) system
following nursing actions? B) Oral opioids supplemented by NSAIDs
A) Maintenance of bed rest to aid healing C) Distraction and relaxation techniques
B) Choosing appropriate splints and functional supplemented by NSAIDs
devices D) A combination of benzodiazepines and topical
C) Administration of beta adrenergic blockers anesthetics
D) Prevention of venous thromboembolism
The nurse caring for a patient who is recovering
A patient is in the acute phase of a burn injury. One from full-thickness burns is aware of the patient's
of the nursing diagnoses in the plan of care is risk for contracture and hypertrophic scarring. How
Ineffective Coping Related to Trauma of Burn can the nurse best mitigate this risk?
Injury. What interventions appropriately address A) Apply skin emollients as ordered after
this diagnosis? Select all that apply. granulation has occurred.
A) Promote truthful communication. B) Keep injured areas immobilized whenever
B) Avoid asking the patient to make decisions. possible to promote healing.
C) Teach the patient coping strategies. C) Administer oral or IV corticosteroids as ordered.
D) Administer benzodiazepines as ordered. D) Encourage physical activity and range of motion
E) Provide positive reinforcement. exercises.

A patient who was burned in a workplace accident While performing a patient's ordered wound care
has completed the acute phase of treatment and for the treatment of a burn, the patient has made a
the plan of care has been altered to prioritize series of sarcastic remarks to the nurse and
rehabilitation. What nursing action should be criticized her technique. How should the nurse best
prioritized during this phase of treatment? interpret this patient's behavior?
A) Monitoring fluid and electrolyte imbalances A) The patient may be experiencing an adverse
B) Providing education to the patient and family drug reaction that is affecting his cognition and
C) Treating infection behavior.
D) Promoting thermoregulation B) The patient may be experiencing neurologic or
psychiatric complications of his injuries.
A burn patient is transitioning from the acute phase C) The patient may be experiencing inconsistencies
of the injury to the rehabilitation phase. The patient in the care that he is being provided.
tells the nurse, ìI can't wait to have surgery to D) The patient may be experiencing anger about
reconstruct my face so I look normal again.î What his circumstances that he is deflecting toward the
would be the nurse's best response? nurse.
A) ìThat's something that you and your doctor will
likely talk about after your scars mature.î A home care nurse is performing a visit to a
patient's home to perform wound care following the
patient's hospital treatment for severe burns. While
interacting with the patient, the nurse should Multiple patients arrive in the emergency
assess for evidence of what complication? department from a house fire. Which patient is a
A) Psychosis priority?
B) Post-traumatic stress disorder A. Patient with erythremic, dry burns over the arms
C) Delirium and a history of taking prednisone
D) Vascular dementia B. Patient with moist blisters over the chest and
who reports pain as 10
A nurse who provides care on a burn unit is C. Patient with dry, black skin on one hand and a
preparing to apply a patient's ordered topical history of diabetes mellitus
antibiotic ointment. What action should the nurse D. Patient with multiple reddened skin areas on the
perform when administering this medication? chest and with high-pitched respiratory sounds
A) Apply the new ointment without disturbing the
existing layer of ointment. A client with burn injury asks the nurse what the
B) Apply the ointment using a sterile tongue term full thickness means. The nurse should
depressor. respond that burns classified as full thickness
C) Apply a layer of ointment approximately 1/16 involve tissue destruction down to which level?
inch thick. a. epidermis
D) Gently irrigate the wound bed after applying the b. dermis
antibiotic ointment. c. subcutaneous tissue
d. internal organs
A client's burn is infected and mafenide
(Sulfamylon) is prescribed. The nurse's knowledge A child has just been admitted to the pediatric burn
about this medication would indicate that which unit. Currently, the child is being evaluated for
organism is involved? burns to his chest and upper legs. He complains of
a. pseudomonas aeruginosa thirst and asks for a drink. What is the most
b. tubercle bacillus appropriate nursing action?
c. Methicillin resistant staphylococcus aureus a. give a small glass of clear liquid
(MRSA) b. give a small glass of a full liquid
d. Candida albicans c. keep the child NPO
d. order a pediatric meal tray with extra liquids
The nurse should expect to apply which type of
ordered antiseptic to a client with a burn wound, A child was admitted to the ED with a thermal burn
once the area has been cleansed with sterile to the right arm and leg. Which assessment by the
saline? nurse requires immediate action?
a. copper containing a. coughing and wheezing
b. silver containing b. bright red skin with small blister on the burn sites
c. biguanide c. thirst
d. acetic acid d. singed hair

The patient sustained a full-thickness burn The condition of a client with extensive third degree
encompassing the entire right arm. What is the best burns begins to deteriorate. The nurse is aware that
indicator an escharotomy achieved its desired which type of shock may occur as a result of
effect? inadequate circulating blood volume that occurs
A. Patient rates the pain at less than 4. with a burn injury?
B. Blood pressure remains above 120/80 mm Hg. a. cardiogenic
C. Right fingers blanch with a 2-second refill. b. distributive
D. Patient maintains full range of motion for the c. hypovolemic
right arm. d. septic
A patient is admitted to the burn center with burns
What is the best method for preventing over his head, neck, chest, back, and left arm and
hypovolemic shock in a client admitted with severe hand after an explosion and fire in his garage. On
burns? admission to the unit, you auscultate wheezes
a. administering dopamine throughout the lung fields. On reassessment, you
b. applying medical antishock trousers notice that the wheezes are gone and the breath
c. infusing i.v. fluids sounds are greatly diminished. Which of the
d. infusing fresh frozen plasma following actions is the most appropriate next step?
a. place the pt in high fowler's position
A client is admitted to a burn intensive care unit b. encourage the pt to cough and auscultate the
with extensive full thickness burns. What should be lungs again
the nurse's initial concern? c. document the results and continue to monitor the
a. fluid status pt's progress
b. risk for infection d. anticipate the need for endotracheal intubation
c. body image and notify the physician
d. level of pain
Fluid and electrolyte shifts that occur during the
A nurse is caring for a client with a new donor site early emergent phase include
that was harvested to treat a burn. The nurse a. adherence of albumin to vascular walls
should position the client to: b. movement of potassium into the vascular space
a. allow ventilation of the site c. sequestering of sodium and water in the
b. make the site dependent interstitial fluid.
c. avoid pressure on the site d. hemolysis of red blood cells from large volumes
d. keep the site fully covered of rapidly administered fluid.

Knowing the most common causes of household To maintain a positive nitrogen balance in a major
fires, which of the following prevention strategies burn, the patient must
would the nurse focus on when teaching about fire a. increase normal caloric intake by about 3 times
safety? b. eat a high-protein, low-fat, high-carbohydrate
a. set how water temp at 140 F diet.
b. use only hardwired smoke detectors c. eat at least 1500 calories per day in small,
c. encourage regular home fire exit drills frequent meals.
d. never permit older adults to cook unattended. d. eat rice and whole wheat for the chemical effect
on nitrogen balance
The injury that is least likely to result in a full
thickness burn is: A patient has 25% TBSA burned from a car fire. His
a. sunburn wounds have been debrided and covered with a
b. scald injury silver-impregnated dressing. Your priority
c. chemical burn intervention for wound care is to:
d. electrical injury a. reapply a new dressing without disturbing the
wound bed
When assessing a pt with a partial thickness burn, b. observe the wound for signs of infection during
the nurse would expect to find (select all that apply) dressing changes
a. blisters c. apply cool compresses for pain relief in between
b. exposed fascia dressing changes
c. exposed muscles d. wash the wound aggressively with soap and
d. intact nerve endings water three times each day.
e. red, shiny, wet appearance
Pain management for the burn patient is most D. One half of the total 24-hour fluid requirement
effective when should be administered in the first 4 hours.
a. opoids are administered on a set schedule
around the clock The nurse is caring for a patient with superficial
b. the patient has as much control over the partial-thickness burns of the face sustained within
management of the pain as possible. the last 12 hours. Upon assessment, the nurse
c. there is flexibility to administer opioids withing a would expect to find which of the following
dosage and frequency range symptoms?
d. painful dressing changes are delayed until the A. Blisters
pt's pain is totally relieved. B. Reddening of the skin
C. Destruction of all skin layers
A therapeutic measure used to prevent D. Damage to sebaceous glands
hypertrophic scarring during the rehabilitative
phase of burn recovery is: The nurse is planning care for a patient with partial-
a. applying pressure garments. and full-thickness skin destruction related to burn
b. repositioning the pt every 2 hours injury of the lower extremities. Which of the
c. performing active ROM at least every 4 hours following interventions would the nurse expect to
d. massaging the new tissue with water based include in this patient's care (select all that apply)?
moisturizers A.Escharotomy
B.Administration of diuretics
C.IV and oral pain medications
A patient has been treated for second- and D.Daily cleansing and debridement
third-degree burns over 30% of his body and is now E.Application of topical antimicrobial agent
ready for discharge. You provide discharge
instructions related to wound care. Which The nurse is caring for a patient with partial- and
statement indicates that the patient understands full-thickness burns to 65% of the body. When
the instructions? planning nutritional interventions for this patient, the
a. I can expect occasional periods of low grade nurse should implement which of the following
fever and can take Tylenol every 4 hours dietary choices?
b. I must wear my jobst elastic garment all day and A. Full liquids only
can only remove it when I'm going to bed. B. Whatever the patient requests
c. I will need to take sponge baths at home to avoid C. High-protein and low-sodium foods
exposing the wounds to unsterile bath water. D. High-calorie and high-protein foods
d. If any healed areas break open, I should cover
them with a sterile dressing and then immediately A patient is admitted to the emergency department
report it." with first- and second-degree burns after being
involved in a house fire. Which of the following
A nurse is caring for a patient with second- and assessment findings would alert you to the
third-degree burns to 50% of the body. The nurse presence of an inhalation injury (select all that
prepares fluid resuscitation based on knowledge of apply)?
the Parkland (Baxter) formula that includes which of A.Singed nasal hair
the following recommendations? B.Generalized pallor
A. The total 24-hour fluid requirement should be C.Painful swallowing
administered in the first 8 hours. D.Burns on the upper extremities
B. One half of the total 24-hour fluid requirement E.History of being involved in a large fire
should be administered in the first 8 hours.
C. One third of the total 24-hour fluid requirement When caring for a patient with an electrical burn
should be administered in the first 4 hours. injury, the nurse should question a health care
provider's order for
A. Mannitol 75 gm IV. C. A 36 year old male with full-thickness burns on
B. Urine for myoglobulin. the front of the left arm.
C. Lactated Ringer's at 25 ml/hr. D. A 10 year old with superficial burns on the right
D. Sodium bicarbonate 24 mEq every 4 hours. leg.

A patient is admitted with first- and second-degree 5. The _____________ layer of the skin helps
burns covering the face, neck, entire right upper regulate our body temperature.
extremity, and the right anterior trunk area. Using A. Epidermis
the rule of nines, the nurse would calculate the B. Dermis
extent of these burns as being C. Hypodermis
A. 9%. D. Fascia
B. 18%.
C. 22.5%. 6. You receive a patient who has experienced a
D. 36%. burn on the right leg. You note the burn contains
small blisters and is extremely pinkish red and
1. A 65 year old male patient has experienced shiny/moist. The patient reports severe pain. You
full-thickness electrical burns on the legs and arms. document this burn as:
As the nurse you know this patient is at risk for the A. 1st Degree (superficial)
following: Select all that apply: B. 2nd Degree (partial-thickness)
A. Acute kidney injury C. 3rd Degree (full-thickness)
B. Dysrhythmia D. 4th Degree (deep full-thickness)
C. Iceberg effect
D. Hypernatremia 7. Based on the depth of the burn in figure 1
E. Bone fractures (picture is above), you would expect to find:
F. Fluid volume overload A. report of sensation to only pressure
B. blanching
2. True or False: A patient who experiences an C. anesthetization to feeling
alkali chemical burn is easier to treat because the D. extreme pain
skin will neutralize the chemical rather than with an
acidic chemical burn. 8. A 58 year old female patient has superficial
partial-thickness burns to the anterior head and
3. As the nurse providing care to a patient who neck, front and back of the left arm, front of the
experienced a full-thickness electrical burn you right arm, posterior trunk, front and back of the right
know to monitor the patient’s urine for: leg, and back of the left leg. Using the Rule of
A. Hemoglobin and myoglobin Nines, calculate the total body surface area
B. Free iron and white blood cells percentage that is burned?
C. Protein and red blood cells A. 63%
D. Potassium and Urea B. 81%
C. 72%
4. Select the patient below who is at MOST risk for D. 54%
complications following a burn:
9. A 30 year old female patient has deep partial
thickness burns on the front and back of the right
A. A 42 year old male with partial-thickness burns and left leg, front of right arm, and anterior trunk.
on the front of the right and left arms and legs. The patient weighs 63 kg. Use the Parkland Burn
B. A 25 year old female with partial-thickness burns Formula: What is the flow rate during the FIRST 8
on the front of the head and neck and front and hours (mL/hr) based on the total you calculated?
back of the torso. A. 921 mL/hr
B. 938 mL/hr
C. 158 mL/hr C. Prep the patient for fasciotomy.
D. 789 mL/hr D. Prep the patient for intubation.
E. Place a pillow under the patient’s neck.
10. A patient has a burn on the back of the torso F. Obtain IV access at two sites.
that is extremely red and painful but no blisters are G. Restrict fluids.
present. When you pressed on the skin it blanches.
You document this as a: 15. A patient arrives to the ER due to experiencing
A. 1st degree (superficial) burn burns while in an enclosed warehouse. Which
B. 2nd degree (partial-thickness) burn assessment findings below demonstrate the patient
C. 3rd degree (full-thickness) burn may have experienced an inhalation injury?
D. 4th degree (deep full-thickness) burn A. Carbonaceous sputum
B. Hair singeing on the head and nose
11. A patient has experienced full-thickness burns C. Lhermitte’s Sign
to the face and neck. As the nurse it is priority to:
A. Prevent hypothermia D. Bright red lips
B. Assess the blood pressure E. Hoarse voice
C. Assess the airway F. Tachycardia
D. Prevent infection
16. You’re providing education to a group of local
12. A patient is in the acute phase of burn firefighters about carbon monoxide poisoning.
management. The patient experienced Which statement is correct about the
full-thickness burns to the perineum and sacral pathophysiology regarding this condition?
area of the body. In the patient’s plan of care, which A. “Patients are most likely to present with cyanosis
nursing diagnosis is priority at this time? around the lips and face.”
B. “In this condition, carbon monoxide binds to the
A. Impaired skin integrity hemoglobin of the red blood cell leading to a
B. Risk for fluid volume overload decrease in the ability of the hemoglobin to carry
C. Risk for infection oxygen to the body.”
D. Ineffective coping C. “Carbon monoxide poisoning leads to a
hyperoxygenated state, which causes
13. The nurse notes a patient has full-thickness hypercapnia.”
circumferential burns on the right leg. The nurse D. “Carbon monoxide binds to the hemoglobin of
would: select all that apply the red blood cell and prevents the transport of
A. Place cold compressions on the burn and carbon dioxide out of the blood, which leads to
elevate the right leg below the heart level poisoning.”
B. Assess the distal pulses in the right extremity
C. Elevate the right leg above the heart level 17. A patient experienced a full-thickness burn 72
D. Place gauze securely around the leg to prevent hours ago. The patient’s vital signs are within
infection normal limits and urinary output is 50 mL/hr. This is
known as what phase of burn management?
14. A patient arrives to the ER with full-thickness A. Emergent
burns on the front and back of the torso and neck. B. Acute
The patient has no spinal injuries but is disoriented C. Rehabilitative
and coughing up black sooty sputum. Vital signs
are: oxygen saturation 63%, heart rate 145, blood 18. What are some patient priorities during the
pressure 80/56, and respiratory rate 39. As the emergent phase of burn management?
nurse you will: A. Fluid volume
A. Place the patient in High Fowler’s positon. B. Respiratory status
B. Prep the patient for escharotomy. C. Psychosocial
D. Wound closure 23. You’re assisting the nursing assistant with
E. Nutrition repositioning a patient with full-thickness burns on
the neck. Which action by the nursing assistant
19. During the emergent phase of burn requires you to intervene?
management, you would expect the following lab A. The nursing assistant elevates the head of the
values: bed above 30 degrees.
B. The nursing assistant places a pillow under the
patient’s head.
A. Low sodium, low potassium, high glucose, low C. The nursing assistant places rolled towels under
hematocrit the patient’s shoulders.
B. High sodium, low potassium, low glucose, high D. The nursing assistant covers the patient with
hematocrit sterile linens.
C. High sodium, high potassium, high glucose, low
hematocrit 24. A patient has full-thickness burns on the front
D. Low sodium, high potassium, high glucose, high and back of both arm and hands. It is nursing
hematocrit priority to:
A. Elevate and extend the extremities
20. A patient is receiving IV Lactated Ringers 950 B. Elevate and flex the extremities
mL/hr post 18 hours after a receiving a severe C. Keep extremities below heart level and extended
burn. The patient urinary output is 20 mL/hr. As the D. Keep extremities level with the heart level and
nurse your next nursing action is to: flexed
A. Increase the IV fluids
B. Continue to monitor the patient 25. A patient has an emergency escharotomy
C. Decrease the IV fluids performed on the right leg. The patient has
D. Notify the physician of this finding full-thickness circumferential burns on the leg.
Which finding below demonstrates the procedure
21. A patient who is being treated for partial was successful?
thickness burns on 60% of the body is now in the A. The patient can move the extremity.
acute phase of burn management. The nurse B. The right foot’s capillary refill is less than 2
assesses the patient for a possible Curling’s Ulcer. seconds.
What signs and symptoms can present with this C. The patient reports a new sensation of extreme
condition? pain.
A. Swelling and pain on the area distal to the burn D. The patient has a positive babinski reflex.
B. Burning, gnawing sensation pain in the stomach
and vomiting 26. Your patient with severe burns is due to have a
C. Dark red or gray sores on the soles of the feet dressing change. You will pre-medicate the patient
D. Difficulty swallowing and gagging prior to the dressing change. The patient has
standing orders for all the medications below.
22. During the acute phase of burn management, Which medication is best for this patient?
what is the best diet for a patient who has A. IM morphine
experienced severe burns? B. PO morphine
A. High fiber, low calories, and low protein C. IV morphine
B. High calorie, high protein and carbohydrate D. Subq morphine
C. High potassium, high carbohydrate, and low
protein 27. After receiving report on a patient receiving
treatment for severe burns, you perform your
D. Low sodium, high protein, and restrict fluids to 1 head-to-toe assessment. On arrival to the patient’s
liter per day room you note the room temperature to be 75’F.
You will:
A. Decrease the temperature by 5-10 degrees to A. Start intravenous fluids.
prevent hyperthermia. B. Check the pulses using a Doppler device.
B. Leave the temperature setting. C. Obtain a complete blood count (CBC).
C. Increase the temperature to a minimum of 85’F. D. Obtain an electrocardiogram (ECG).

28. You are about to provide care to a patient with 3. A 40-year-old male client who was burned was
severe burns. You will don: admitted under your care. Assessment reveals he
A. gloves has crackles, respiratory rate of 40/min, and is
B. goggles coughing up blood-tinged sputum. What action will
C. gown the nurse take first?
D. N-95 mask A. Administer digoxin
E. surgical mask B. Perform chest physiotherapy
F. shoe covers C. Monitor urine output
G. hair cover D. Place the client in an upright position

29. While collecting a medical history on a patient 4. How will the nurse position a client with a burn
who experienced a severe burn, which statement wound to the posterior neck to prevent
by the patient’s family member requires nursing contractures?
intervention?
A. “He takes medication for glaucoma”. A. Have the client turn the head from side to side.
B. “I think it has been 10 years or more since he B. Keep the client in a supine position without the
had a tetanus shot.” use of pillows.
C. “He was told he had COPD last year.” C. Keep the client in a semi-Fowler's position with
D. “He smokes 2 packs of cigarettes a day.” her or his arms elevated.
D. Place a towel roll under the client's neck or
30. A patient is presenting with bright red lips, shoulder.
headache, and nausea. The physician suspects
carbon monoxide poisoning. As the nurse, you 5. On assessment, the nurse notes that the client
know the patient needs: has burns inside the mouth and is wheezing.
A. Oxygen nasal cannula 5-6 Liters Several hours later, the wheezing is no longer
B. 100% oxygen via non-rebreather mask heard. What is the nurse’s next action?
A. Documenting the findings
C. Continuous Bipap B. Loosening any dressings on the chest
D. Venturi mask 6 L oxygen C. Raising the head of the bed
D. Preparing for intubation
1. A 23-year-old male client who has had a
full-thickness burn is being discharged from the 6. Ten hours after the client with 50% burns is
hospital. Which information is most important for admitted, her blood glucose level is 142 mg/dL.
the nurse to provide prior to discharge? What is the nurse’s best action?
A. How to maintain home smoke detectors A. Documents the finding
B. Joining a community reintegration program B. Obtains a family history of diabetes
C. Learning to perform dressing changes C. Repeats the glucose measurement
D. Options available for scar removal D. Stop IV fluids containing dextrose

2. A client who is admitted after a thermal burn 7. A 32-year-old client, who is on an anticoagulant
injury has the following vital signs: blood pressure, medication due to a history of deep vein
70/40; heart rate, 140 beats/min; respiratory rate, thrombosis, arrives at the emergency department
25/min. He is pale in color and it is difficult to find after accidentally spilling a pot of boiling water
pedal pulses. Which action will the nurse take first? mixed with hot oil on their right arm while
attempting to deep-fry food at home. The client is in B. Ascertains the time food or liquid was last
significant pain and reports that the burn occurred consumed
approximately 30 minutes ago. Upon assessment, C. Auscultates breath sounds over the trachea and
the nurse observes a large burned area on the mainstem bronchi
client’s right arm with a mix of pink and mottled red D. Measures abdominal girth and auscultates
areas, blisters, and signs of active bleeding from bowel sounds
the blistered areas. The burned area is very painful Incorrect
to touch and the client is visibly anxious.
12. A 22-year-old female client with a full-thickness
Based on the assessment findings and the client’s burn is being discharged to home after a month in
medical history, how will the nurse categorize this the hospital. Her wounds are minimally opened and
burn injury? she will be receiving home care. Which nursing
A. Full-thickness diagnosis has the highest priority?
B. Partial-thickness superficial A. Acute Pain
C. Partial-thickness deep B. Deficient Diversional Activity
D. Full-thickness deep C. Impaired Adjustment
D. Imbalanced Nutrition: Less than Body
8. The client has burns on both legs. These areas Requirements
appear white and leather-like. No blisters or
bleeding are present, and there is just a “small 13. The client with a new burn injury asks the nurse
amount of pain.” How will the nurse categorize this why he is receiving intravenous cimetidine
injury? (Tagamet). What is the nurse’s best response?
A. Full-thickness
B. Partial-thickness superficial A. "Tagamet will stimulate intestinal movement."
C. Partial-thickness deep B. "Tagamet can help prevent hypovolemic shock."
D. Superficial C. "This will help prevent stomach ulcers."
D. "This drug will help prevent kidney damage."
9. The client has experienced an electrical injury of Incorrect
the lower extremities. Which are the priority
assessment data to obtain from this client? 14. A 12-year-old male with facial burns asks the
A. Current range of motion in all extremities nurse if he will ever look the same. Which response
B. Heart rate and rhythm is best for the nurse to provide?
C. Respiratory rate and pulse oximetry reading A. "With reconstructive surgery, you can look the
D. Orientation to time, place, and person same."
B. "We can remove the scars with the use of a
10. A 35-year-old male client was admitted due to pressure dressing."
severe burns around his right hip. Which position is C. "You will not look exactly the same."
most important to use to maintain the maximum D. "You shouldn't start worrying about your
function of this joint? appearance right now."
A. Hip maintained in 30-degree flexion
B. Hip at zero flexion with leg flat 15. The client with open burn wounds begins to
C. Knee flexed at 30-degree angle have diarrhea. The client is found to have a
D. Leg abducted with a foam wedge below-normal temperature, with a white blood cell
count of 4000/mm3. Which is the nurse’s best
11. The client who is burned is drooling and having action?
difficulty swallowing. Which action will the nurse
take first? A. Continuing to monitor the client
A. Assesses level of consciousness and pupillary B. Increasing the temperature in the room
reactions C. Increasing the rate of the intravenous fluids
D. Preparing to do a workup for sepsis 1.9 Poisoning and Antidotes

16. The family of a client who has been burned 1.10 SARS
asks at what point the client will no longer be at
greater risk for infection. What is the nurse’s best 1.11 ARDS
response? 1.) You’re providing care to a patient who is being
A. "As soon as he finishes his antibiotic treated for aspiration pneumonia. The patient is on
prescription." a 100% non-rebreather mask. Which finding below
B. "As soon as his albumin level returns to is a HALLMARK sign and symptom that the patient
normal." is developing acute respiratory distress syndrome
C. "When fluid remobilization has started." (ARDS)?
D. "When the burn wounds are closed." A. The patient is experiencing bradypnea.
B. The patient is tired and confused.
17. The nurse is conducting a home safety class. It C. The patient’s PaO2 remains at 45 mmHg.
is most important for the nurse to include which D. The patient’s blood pressure is 180/96.
information in the teaching plan?
A. Have chimneys swept every 2 years. 2. You’re teaching a class on critical care concepts
B. Keep a smoke detector in each bedroom. to a group of new nurses. You’re discussing the
C. Use space heaters instead of gas heaters. topic of acute respiratory distress syndrome
D. Use carbon monoxide detectors only in the (ARDS). At the beginning of the lecture, you assess
garage. the new nurses understanding about this condition.
Incorrect Which statement by a new nurse demonstrates he
understands the condition?
18. The nurse provides wound care for a client 48 A. “This condition develops because the exocrine
hours after a burn injury. To achieve the desired glands start to work incorrectly leading to thick,
outcome of the procedure, which nursing action will copious mucous to collect in the alveoli sacs.”
be carried out first? B. “ARDS is a pulmonary disease that gradually
A. Applies silver sulfadiazine (Silvadene) ointment causes chronic obstruction of airflow from the
B. Covers the area with an elastic wrap lungs.”
C. Places a synthetic dressing over the area C. “Acute respiratory distress syndrome occurs due
D. Removes loose nonviable tissue to the collapsing of a lung because air has
accumulated in the pleural space.”
19. The nurse should teach the community that a D. “This condition develops because alveolar
minor burn injury could be caused by what common capillary membrane permeability has changed
occurrence? leading to fluid collecting in the alveoli sacs.”
A. Chimney sweeping every year
B. Cooking with a microwave oven 3. During the exudative phase of acute respiratory
C. Use of sunscreen agents distress syndrome (ARDS), the patient’s lung cells
D. Use of space heaters that produce surfactant have become damaged. As
the nurse you know this will lead to?
20. The nurse uses topical gentamicin sulfate A. bronchoconstriction
(Garamycin) on a client’s burn injury. Which B. atelectasis
laboratory value will the nurse monitor? C. upper airway blockage
A. Creatinine D. pulmonary edema
B. Red blood cells
C. Sodium 4. A patient has been hospitalized in the ICU for a
D. Magnesium level near drowning event. The patient’s respiratory
function has been deteriorating over the last 24
hours. The physician suspects acute respiratory
distress syndrome. A STAT chest x-ray is ordered. C. PaO2 80 mmHg
What finding on the chest x-ray is indicative of D. PaCO2 38 mmHg
ARDS?
A. infiltrates only on the upper lobes 9. You are caring for a patient with acute respiratory
B. enlargement of the heart with bilateral lower lobe distress syndrome. As the nurse you know that
infiltrates prone positioning can be beneficial for some
C. white-out infiltrates bilaterally patients with this condition. Which findings below
D. normal chest x-ray indicate this type of positioning was beneficial for
your patient with ARDS?
5. You’re providing care to a patient who was just A. Improvement in lung sounds
transferred to your unit for the treatment of ARDS. B. Development of a V/Q mismatch
The patient is in the exudative phase. The patient is C. PaO2 increased from 59 mmHg to 82 mmHg
ordered arterial blood gases. The results are back. D. PEEP needs to be titrated to 15 mmHg of water
Which results are expected during this early phase
of acute respiratory distress syndrome that 10. A patient is experiencing respiratory failure due
correlates with this diagnosis? to pulmonary edema. The physician suspects
A. PaO2 40, pH 7.59, PaCO2 30, HCO3 23 ARDS but wants to rule out a cardiac cause. A
B. PaO2 85, pH 7.42, PaCO2 37, HCO3 26 pulmonary artery wedge pressure is obtained. As
C. PaO2 50, pH 7.20, PaCO2 48, HCO3 29 the nurse you know that what measurement
D. PaO2 55, pH 7.26, PaCO2 58, HCO3 19 reading obtained indicates that this type of
respiratory failure is NOT cardiac related?
6. Which patient below is at MOST risk for A. >25 mmHg
developing ARDS and has the worst prognosis? B. <10 mmHg
A. A 52-year-old male patient with a pneumothorax. C. >50 mmHg
B. A 48-year-old male being treated for diabetic D. <18 mmHg
ketoacidosis.
C. A 69-year-old female with sepsis caused by a 11. You’re precepting a nursing student who is
gram-negative bacterial infection. assisting you care for a patient on mechanical
D. A 30-year-old female with cystic fibrosis. ventilation with PEEP for treatment of ARDS. The
student asks you why the PEEP setting is at 10
7. As the nurse you know that acute respiratory mmHg. Your response is:
distress syndrome (ARDS) can be caused by direct A. “This pressure setting assists the patient with
or indirect lung injury. Select below all the breathing in and out and helps improve air flow.”
INDIRECT causes of ARDS: B. “This pressure setting will help prevent a
A. Drowning decrease in cardiac output and hyperinflation of the
B. Aspiration lungs.”
C. Sepsis C. “This pressure setting helps prevent fluid from
D. Blood transfusion filling the alveoli sacs.”
E. Pneumonia D. “This pressure setting helps open the alveoli
F. Pancreatitis sacs that are collapsed during exhalation.”

8. A patient is on mechanical ventilation with PEEP The nurse is caring for a client with suspected
(positive end-expiratory pressure). Which finding acute respiratory distress syndrome​(ARDS). Which
below indicates the patient is developing a symptom of ARDS should the nurse anticipate will
complication related to their therapy and requires appear within 24 to 48 hours after the initial​insult?
immediate treatment? (Select all that​apply.)
A. HCO3 26 mmHg Shortness of breath
Rapid breathing
B. Blood pressure 70/45 Fluid imbalance
Chest​x-ray clear of infiltrates "​ We will administer medication to help him sleep
Arterial blood gases varying from normal limits through the intubation and as needed while he is on
the​ventilator."
A client is diagnosed with acute respiratory distress ​"Don't worry; intubation and using a ventilator​
syndrome​ (ARDS). The​ client's spouse asks the doesn't hurt at​all."
nurse what caused ARDS. Which etiology of ​"He may feel a little​nervous, but he will get used to
indirect injury to the lungs should the nurse include the ventilator soon​enough."
in the​response? (Select all that​apply.) ​"He will actually feel better and may even like the
Smoke inhalation ventilator because he will be able to breathe more​
Systemic sepsis comfortably."
Fat embolism
Multiple blood transfusions A pregnant woman presents with worsening
Pancreatitis dyspnea and tachypnea since being diagnosed with
pneumonia last week. The client is currently at 28
The nurse is caring for a client who had an episode weeks of gestation and asks the nurse if she will
of​ near-drowning 5 days ago. This​ morning, the need to deliver early. Which is the best reply by the​
nurse noted rhonchi in the lower lung lobes on nurse?
auscultation. Which action by the nurse is best​?
Documenting the findings as normal "​ Yes, we will need to delivery your baby as soon as​
Preparing for intubation possible."
Notifying the healthcare provider ​"It's hard to know right​now."
Monitoring vital signs and oxygen saturation every ​"There's no need to deliver your baby early. The
2 hours baby is safer in your​womb."
​"We'll monitor the baby very closely and plan for
delivery if needed for the​baby's safety."
A client is brought into the emergency department
after aspirating on pureed foods at the​ long-term
care facility. The nurse knows that which A pregnant woman has been diagnosed with acute
physiologic change can trigger acute respiratory respiratory distress syndrome​ (ARDS) and
distress​syndrome? prescribed corticosteroids. The woman asks the
Release of chemical mediators nurse if it is safe to take this medication. Which is
Increased surfactant production the best reply by the​nurse?
Destruction of extracellular platelets ​"Corticosteroids are safe to take during pregnancy
Intracellular edema in some​cases, and you need the​medicine."
​"Let's call the doctor and pharmacist to double
The nurse is caring for a client with respiratory check the safety profile of these​drugs."
acidosis secondary to​ end-stage acute respiratory ​"That is an​error; you​shouldn't take corticosteroids
distress syndrome​(ARDS). Which result should the during​pregnancy."
nurse anticipate on the arterial blood​gas? ​"If the doctor prescribed the​ medication, I'm sure
High PaO2 and low PaCO2 that​it's fine to take​it."
Low PaO2 and low PaCO2
Low PaO2 and high PaCO2 The nurse is preparing a program about acute
High PaO2 and high PaCO2 respiratory distress syndrome​ (ARDS) in the older
adult and plans to explain why this population is at
A pediatric client is not responding to treatment for greater risk for the disease. Which topic should be
acute respiratory distress syndrome​ (ARDS) and included in the​discussion? (Select all that​apply.)
requires intubation. The mother is anxious that the Poor muscle strength
procedure will hurt. Which is the best response by Lower functional residual capacity
the​nurse? Loss of elastic tissue
Increased chest diameter The nurse is caring for a client who has been using
Higher sedation requirements mechanical ventilation for several months after an
episode of sepsis and acute respiratory distress
The nurse is caring for an older adult client who syndrome​ (ARDS). Which ventilator setting should
may need to be intubated. The family asks about the nurse anticipate the healthcare provider
the potential risks. Which is the best reply by the​ ordering for​weaning?
nurse? Synchronized intermittent mandatory ventilation​
(SIMV)
"​ There are no risks to​intubation." Positive​end-expiratory pressure​(PEEP)
​"The doctor feels that intubation is the best option ​Assist-control mode ventilation​(ACMV)
for your family​member." Bilevel ventilation​(BIPAP)
​"Older adults may be more likely to experience
injury to the airway during​intubation." During assessment of a client with acute respiratory
​"Intubation and mechanical ventilation are not​ distress syndrome​ (ARDS), the nurse notes an
long-term solutions. They just allow the lungs to​ oxygen saturation of​78% and a respiratory rate of
rest." 28​ breaths/min. The nurse notifies the healthcare
provider and should prepare for intubation using
The nurse is caring for an adult who is diagnosed which type of​airway?
with acute respiratory distress syndrome​ (ARDS) Tracheostomy
after a​ near-drowning episode last week. Which Endotracheal tube
type of medication should the nurse anticipate the Oropharyngeal airway
provider ordering to help open​alveoli? Nasopharyngeal airway
Inhaled nitric oxide
Surfactant therapy The nurse is assessing an older adult client with
Nonsteroidal​anti-inflammatory drugs​(NSAIDs) acute respiratory distress syndrome​(ARDS). Which
Corticosteroids assessment finding indicates an early sign of
hypoxemia for this​client? (Select all that​apply.)
The nurse is caring for a client with acute Agitation
respiratory distress syndrome​ (ARDS) who needs Tachypnea
an artificial airway to assist in maintaining an open Confusion
airway. Which airway will the nurse plan to Anxiety
reposition every 8 hours while providing​care? Dyspnea
Endotracheal
Nasopharyngeal The nurse is providing care to a client with acute
Oropharyngeal respiratory distress syndrome​ (ARDS). Which
Tracheostomy independent intervention should the nurse prepare
to perform for this​client? (Select all that​apply.)
A client who is diagnosed with acute respiratory Maintain the head of the bed at 30 degrees.
distress syndrome​ (ARDS) requires mechanical
ventilation. Which ventilator mode should the nurse Recommend a prone position to facilitate
expect to implement to promote pressure oxygenation.
throughout the respiratory​cycle? Prescribe analgesia for pain.
Sensitivity Order a Foley catheter to monitor urine output.
Positive​end-expiratory pressure​(PEEP) Auscultate heart and lung sounds.
Flow rate
Tidal volume​(TV) The nurse is planning care for a client with acute
respiratory distress syndrome​ (ARDS). Which
independent nursing intervention should the nurse
include in the care of this​ client? (Select all that​
apply.)
Maintain the head of the bed at less than 30
degrees.
Obtain a sputum culture.
Prescribe surfactant therapy.
Position the client in a prone position for 60 minutes
five times a day.
Suction the airway as needed.

The nurse is developing a plan of care for an older


adult client who presents with hypoxemia and
agitation. What nursing diagnosis is the priority for
this​client?
​Confusion, Risk for Acute
​Coping, Ineffective
Anxiety
​Pain, Acute

The nurse is caring for a client who is in the


process of weaning off of mechanical ventilation.
Which assessment finding should the nurse report
to the healthcare​provider? (Select all that​apply.)
Abdominal breathing
Oxygen saturation level of​98%
Respiratory rate of 18​beats/min
Pallor
Agitation

2.1 CPR/Basic Life Support

2.2 First Aid Training

You might also like